Chapter 10: Neurology
Neurology
Dr Khaled Magraby MRCP Notes
Chapter 10: Neurology
Transient Ischaemic Attack (TIA)
NICE issued updated guidelines relating to stroke and TIA.
They advocated the use of the ABCD2 prognostic score for risk stratifying patients
who've had a suspected TIA:
Criteria Points
A Age >= 60 years 1
B Blood pressure >= 140/90 mmHg 1
C Clinical features:
- Unilateral weakness. 2
- Speech disturbance, no weakness. 1
D Duration of symptoms:
- > 60 minutes. 2
- 10-59 minutes. 1
D Patient has Diabetes 1
TOTAL 7
This gives a total score ranging from 0 to 7.
People who have had a suspected TIA who are at a higher risk of stroke (that is, with
an ABCD2 score of ≥ 4 should have:
1) Aspirin (300 mg daily) started immediately.
2) Specialist assessment and investigation within 24 hours of onset of
symptoms.
3) Measures for secondary prevention introduced as soon as the diagnosis is
confirmed, including discussion of individual risk factors.
If the ABCD2 risk score is ≤ 3:
Specialist assessment within 1 week of symptom onset, including decision
on brain imaging.
If vascular territory or pathology is uncertain, refer for brain imaging.
Dr Khaled Magraby MRCP Notes Neurology 946
Chapter 10: Neurology
People with crescendo TIAs (2 or more episodes in a wk) should be ttt as being at
high risk of stroke, even though they may have an ABCD2 score of 3 or below.
Antithrombotic therapy:
Clopidogrel is recommended first-line (as for patients who've had a stroke).
Aspirin + dipyridamole should be given to patients who cannot tolerate
clopidogrel.
These guidelines may change following the CHANCE study (NEJM 2013;
369:11). This study looked at giving high-risk TIA patients aspirin +
clopidogrel for the first 90 days compared to aspirin alone. 11.7% of
aspirin only patients had a stroke over 90 days compared to 8.2% of dual
antiplatelet patients.
Antiplatelets:
TIA: Clopidogrel
Ischaemic stroke: Clopidogrel
Stroke: management
The Royal College of Physicians (RCP) and NICE guidelines:
Selected points relating to the management of acute stroke include:
Blood glucose, hydration, oxygen saturation and temperature should be
maintained within normal limits
Blood pressure should not be lowered in the acute phase unless there
are complications e.g. Hypertensive encephalopathy.
Aspirin 300mg orally or rectally should be given as soon as possible if a
haemorrhagic stroke has been excluded.
With regards to atrial fibrillation, the RCP state: 'anticoagulants should not
be started until brain imaging has excluded haemorrhage, and usually
not until 14 days have passed from the onset of an ischaemic stroke'.
If the cholesterol is > 3.5 mmol/l patients should be commenced on a statin.
Many physicians will delay treatment until after at least 48 hours due to the
risk of haemorrhagic transformation.
Dr Khaled Magraby MRCP Notes Neurology 947
Chapter 10: Neurology
Thrombolysis:
Thrombolysis should only be given if:
It is administered within 4.5 hours of onset of stroke symptoms (the
thrombolytic window) (unless as part of a clinical trial).
Haemorrhage has been definitively excluded (i.e. Imaging has been
performed).
Alteplase (tPA) is currently recommended by NICE.
Stroke thrombolysis with tPA >> only consider if less than 4.5 hours and
haemorrhage excluded.
The National Institute of Neurological Disorders and Stroke (NINDS) issued a
protocol with inclusion and exclusion criteria for tPA:
Their inclusion criteria were:
1) Age over 18
2) Clinical diagnosis of acute ischaemic stroke
3) Known time of onset
4) CT scan consistent with diagnosis, and
5) Treatment can be given within 180 minutes (though some physicians treat
after this period).
Their exclusion criteria included:
1) Intracranial haemorrhage on CT scan
2) Symptoms minor or improving
3) Active bleeding at any site
4) Gastrointestinal bleed in the last 21 days
5) Major surgery in last 14 days
6) History of intracranial bleed
7) Serious head injury in last 3 months
8) Pregnancy, or
9) Active pancreatitis.
Secondary prevention:
NICE also published a technology appraisal in 2010 on the use of clopidogrel and
dipyridamole for prevention of further OVE (occlusive vascular events).
Recommendations from NICE include:
Clopidogrel is now recommended by NICE ahead of combination use of
aspirin plus dipyridamole modified release (MR) in people who have had an
ischaemic stroke.
Dr Khaled Magraby MRCP Notes Neurology 948
Chapter 10: Neurology
Aspirin plus dipyridamole MR is now recommended after an ischaemic stroke
only if clopidogrel is contraindicated or not tolerated, but treatment is no
longer limited to 2 years' duration (i.e. it will be lifelong).
Dipyridamole MR alone is recommended after an ischaemic stroke only if
aspirin or clopidogrel are contraindicated or not tolerated, again with no limit
on duration of treatment. (i.e. it will be lifelong).
Secondary prevention of OVE:
Clopidogrel
Aspirin plus dipyridamole MR
Dipyridamole MR alone
Dipyridamole is not to be used in acute phase.
EX: Pt with ischemic stroke: at discharge from hospital after 14 days >> he should
receive: (Clopidogrel + Statin If the cholesterol is > 3.5).
With regards to carotid artery endarterectomy:
Current UK guidelines recommend endarterectomy for symptomatic patients
with greater than 70% stenosis, based on the North American Symptomatic
Carotid Endarterectomy Trial (NASCET) which showed clear benefit. The
endarterectomy should be performed as soon as the patient is fit for
surgery, preferably within 2 weeks of a TIA.
The benefit is marginal for symptomatic patients with 50-69% stenosis, but
may be greater in male patients. NICE recommends these patients are also
considered for endarterectomy.
There is significantly less benefit for asymptomatic patients, even those with
greater than 60% stenosis.
Patients with less than 50% stenosis should not be considered for carotid
surgery.
Recurrent stenosis can occur in 1-20% of patients following endarterectomy,
and re-operation is needed in 1-3% of cases.
All patients with suspected non-disabling stroke or TIA who are considered as
candidates for carotid endarterectomy should have carotid imaging within 1
week.
Indications for carotid endarterectomy:
1) Symptomatic patients with greater than 70% stenosis (NASCET).
2) Symptomatic patients with 50-69% stenosis.
3) Asymptomatic patients with greater than 60% stenosis.
Dr Khaled Magraby MRCP Notes Neurology 949
Chapter 10: Neurology
In general,
1) Symptomatic patients with greater than 50% stenosis, and
2) Healthy asymptomatic patients with greater than 60% stenosis, warrant
consideration for carotid endarterectomy.
Carotid stenting is increasingly being used as an alternative to endarterectomy.
This is a less invasive revascularisation strategy, and uses an embolic protection
device. There seems to be a similar early risk of death or stroke, and similar long-
term benefits. Risk is higher in elderly patients, possibly due to vascular tortuosity
and calcification. The procedure is currently indicated in selected cases, such as
restenosis.
Carotid artery dissection
The two commonest causes of young onset stroke (less than 40 years) are
cardio embolism and carotid artery dissection.
75% of carotid dissections affect the internal carotid artery (that is, extracranially),
and may be related to neck trauma or manipulation, although the cause is often
difficult to identify.
The classic triad of symptoms of carotid dissection are:
1) Unilateral (ipsilateral) headache
2) Ipsilateral Horner's syndrome and
3) Contralateral hemisphere signs (aphasia, neglect, visual disturbance,
and hemiparesis).
Headache is commonly ipsilateral to the side of the carotid dissection, and
recurrence of the headache suggests extension or recurrence of the dissection.
The Horner's syndrome is caused by compression of the ascending sympathetic
supply within the carotid sheath, and results in ptosis and miosis.
Anhydrosis is classically not present as the sympathetic supply to the sweat glands
is along the external carotid plexus and is therefore spared.
Ischaemic neurological features (transient or completed strokes) are found in 30-
80% of patients presenting with carotid artery dissection.
Diagnosed by Contrast arteriography of the neck vessels
Management is aimed at preventing cerebral infarction, and is similar to that of acute
stroke. Stenting can be used if there is ongoing ischaemia.
EX: A 21-year-old female presented with a sudden onset of left sided head and
neck pain.
Dr Khaled Magraby MRCP Notes Neurology 950
Chapter 10: Neurology
24 hours later she presents with sudden onset of right hemiparesis, facial
weakness and homonymous hemianopia and left Horner's syndrome.
A CT brain showed a left middle cerebral artery territory infarction.
The most likely diagnosis? Lt Carotid artery dissection
NB: Migrainous stroke usually affects the posterior circulation (posterior cerebral
artery territory is the commonest).
NB: A thrombotic event resulting from cardio embolism or Antiphospholipid
syndrome would usually only affect intracranial vessels and therefore a Horner's
syndrome would be unusual.
Neurosurgical intervention in acute ischaemic stroke:
1) Patients who are under 60 years of age with large cerebral infarctions
arising in the MCA territory should be considered for decompressive
hemicraniectomy which is removing part of the skull in order to reduce ICP
and should be carried out within 48 hours of the index event.
2) A massive cerebellar infarction or evidence of hydrocephalus or
brainstem compression.
Stroke by anatomy
Site of the lesion Associated effects
Anterior cerebral artery (ACA) -Contralateral hemiparesis and sensory
loss, lower extremity > upper.
-Disconnection syndrome (Akinetic mute
patient).
Middle cerebral artery (MCA) -Contralateral hemiparesis and sensory
loss, upper extremity > lower.
-Contralateral homonymous hemianopia.
-Aphasia (Wernicke’s).
-Gaze abnormalities.
Dr Khaled Magraby MRCP Notes Neurology 951
Chapter 10: Neurology
Site of the lesion Associated effects
Posterior cerebral artery (PCA) - Pure hemisensory loss
- Contralateral homonymous hemianopia
with macular sparing.
-Visual agnosia.
- Disorders of reading (alexia, dyslexia)
- Disconnection syndrome.
Weber's syndrome (branches of -Ipsilateral CN III palsy.
the PCA that supply the midbrain) -Contralateral weakness.
Posterior inferior cerebellar artery - Ipsilateral cerebellar: Ataxia, nystagmus.
(= lateral medullary syndrome),
(=Wallenberg syndrome) - Ipsilateral signs of Horner's syndrome.
- Ipsilateral loss of corneal reflex
- Ipsilateral facial loss of pinprick
sensation
- Contralateral signs of spinothalamic
sensory loss of the limb/torso pain and
temperature loss.
Anterior inferior cerebellar artery Symptoms are similar to Wallenberg's
(lateral pontine syndrome) (see above), but add:
Ipsilateral: facial paralysis and deafness
Pontine -6 th nerve palsy: horizontal gaze palsy.
-7 th nerve palsy
-Contralateral hemiparesis.
Retinal/ophthalmic artery Amaurosis fugax
Dr Khaled Magraby MRCP Notes Neurology 952
Chapter 10: Neurology
Site of the lesion Associated effects
Basilar artery 'Locked-in' syndrome: Pt is awake but is
unable to respond in anyway except by
vertical gaze and blinking (lesion is in
ventral pons).
Lacunar strokes:
Present with either isolated hemiparesis, hemisensory loss or hemiparesis
with limb ataxia.
Strong association with HTN
Common sites include the basal ganglia, thalamus and internal capsule.
Lateral medullary syndrome (Wallenberg's syndrome)
It occurs following occlusion of the posterior inferior cerebellar artery (PICA).
Cerebellar features: Ataxia and nystagmus.
Brainstem features:
1) Ipsilateral: dysphagia, facial numbness, cranial nerve palsy e.g. Horner's
2) Contralateral: limb sensory loss (pyramidal tract signs).
Lateral medullary syndrome >> PICA lesion >> Ipsilateral Cerebellar signs +
Ipsilateral Horner's + Contralateral limb sensory loss.
Brown-Sequard syndrome
It is a loss of sensation and motor function that is caused by the lateral hemisection
of the spinal cord.
Ipsilateral signs include pyramidal weakness and dorsal column dysfunction (joint
position and light touch) and contralateral signs include spinothalamic dysfunction
(pinprick and temperature).
Features:
1) Ipsilateral loss of fine touch, vibration and proprioception.
2) Ipsilateral hyperreflexia and extensor planter reflex.
3) Contralateral loss of pain and temperature sensation.
4) Segmental anaesthesia at the level of the lesion.
Dr Khaled Magraby MRCP Notes Neurology 953
Chapter 10: Neurology
Complete syndrome picture is rare and many patient may only exhibit some features.
Causes:
Trauma is the common cause.
Demyelination due to multiple sclerosis.
Any lateral cord lesion (Ischaemia, haemorrhage, granuloma, tumour etc.
Syringomyelia
Overview:
Development of cavity (syrinx) within the spinal cord that produce
progressive myelopathy.
Symptoms begins insidiously in adolescence or early adulthood, progress
irregularly, and may undergo spontaneous arrest for several years; most
patients acquire a cervical-thoracic scoliosis.
If extends into medulla then termed syringobulbia
Strongly associated (> 50%) with the Arnold-Chiari malformation.
Features:
May be asymmetrical initially.
Slowly progressives, possibly over years.
Motor: wasting and weakness of arms.
Sensory: Spinothalamic sensory loss (pain and temperature).
Loss of reflexes, bilateral upgoing plantars.
Also seen: Horner's syndrome.
Syringomyelia >> Spinothalamic sensory loss (pain and temperature).
EX: A 35-year-old woman presents with pains in the right arm. She has wasting
and weakness of the intrinsic muscles of the right hand, absent tendon reflexes in
the right arm and impaired pinprick sensation in the right hand and forearm.
The most likely diagnosis >>> Syringomyelia
Dr Khaled Magraby MRCP Notes Neurology 954
Chapter 10: Neurology
Localization of the lesion:
At syrinx (there is anterior horn cell involvement) → LMNL weakness.
At central decussating fibers (spinothalamic tract) → dissociated sensory
loss with late development of neuropathic arthropathy.
At corticospinal tracts below the level of the syrinx → spastic paraplegia.
MRI is the investigation of choice
Myelography used to confirm the diagnosis but was associated with more
deterioration.
Transverse myelitis (TM)
It is an inflammatory lesion that can affects the cord.
Constitutional symptoms such as headache and fever are common as is pain.
Signs are indistinguishable from those caused by cord compression and again all
sensory aspects are equally affected with no sparing of proprioception.
The clinical signs are caused by an interruption in ascending and descending
pathways in the transverse plane of the spinal cord.
A sensory level is characteristic.
Midline or dermatomal neuropathic pain can be present.
Urinary incontinence or retention, bowel incontinence or constipation, and sexual
dysfunction are common but vary in severity.
These signs develop over hours to days, and are usually bilateral.
There are a variety of causes, but it most often occurs as an autoimmune
phenomenon after an infection or vaccination, or as a result of direct infection, an
underlying systemic autoimmune disease, or an acquired demyelinating disease. For
a significant proportion of cases no cause is found.
MRI is indicated to rule out the presence of structural lesions, and determine the
presence of myelitis which enhances with gadolinium in the acute phase. There
may be more than one area of myelitis, and the lesions usually span at least two
vertebral segments. In the acute phase the MRI may be normal.
Treatment in the acute phase aims to halt the progression and initiate resolution of
the inflammatory cord lesion. High doses Corticosteroids IV are first line. Plasma
exchange can be given to those who fail to respond. Patients with demyelinating
disease can be started on long term immunosuppression.
Dr Khaled Magraby MRCP Notes Neurology 955
Chapter 10: Neurology
The prognosis is highly variable, and improvement can take 3 months and longer
to develop.
50% to 70% of patients have partial or complete recovery
A rapidly progressive course, severe weakness, hypotonia and areflexia are
predictors of poor prognosis.
Subclavian Steal Syndrome
It is associated with retrograde flow in the vertebral artery due to proximal subclavian
artery stenosis.
Neurological symptoms are precipitated by vigorous exercise with the arm above the
head, such as painting a wall.
Diagnosis is often confused with TIA or epilepsy.
Duplex US and MRA are the investigations of choice.
TTT: Endarterectomy and stenting.
Subacute Combined Degeneration of spinal cord (SCD)
It is also known as Lichtheim’s disease.
It refers to degeneration of the posterior and lateral columns of the spinal cord as
a result of Vit B12 deficiency (most common), Vit E deficiency or Friedrich’s ataxia.
It is usually associated with pernicious anaemia.
Features:
Patchy losses of myelin in the dorsal and lateral columns.
Present with progressive weakness of legs, arms, trunk, tingling and
numbness.
Visual and mental changes may also be present.
Bilateral spastic paresis with diminished pressure, vibration and touch
sense.
Positive Babinski sign may be seen.
Extensor plantars with absent ankle reflexes
Prolonged deficiency of Vit B12 (>3 months) leads to irreversible nervous system
damage.
If someone is deficient on Vit B12 and folic acid, the Vit B12 deficiency should be
treated first to avoid precipitating SCD of the cord.
Dr Khaled Magraby MRCP Notes Neurology 956
Chapter 10: Neurology
TTT with Vit B12 results in partial to full recovery, depending on the duration and
extent of neurodegeneration.
Absent ankle jerks
Absent ankle jerks may occur in conditions associated with neuropathy:
1) B12 deficiency,
2) SLE,
3) Tabes dorsalis (Syphilis)
4) Cerebrotendinous xanthomatosis
Cerebrotendinous xanthomatosis is an inherited condition, associated with
accumulation of cholesterol in tissues including brain, peripheral nerve and tendons
which produces a clinical picture of:
Early onset dementia
Gait ataxia
Loss of vibration sense
Cataracts
Large tendon xanthomata.
There is a deficiency in sterol storage, and diagnosis is based on high serum
(and tendon) cholesTANOL. BUT Serum cholesterol may be normal or low.
It is eminently treatable by the oral chenodeoxycholic acid.
EX: A 39-year-old woman is found to have absent ankle jerks and gait
disturbance >>> so should check: ANA, B12, VDRL, Cholestanol levels
Extensor plantars and absent ankle jerks (Lesions that produce both UMNL
and LMNL)
Typically caused by lesion producing both UMNL (extensor plantars) and LMNL
(absent ankle jerk) signs.
Causes:
1) SCD (Subacute combined degeneration of the cord)
2) MND (Motor neuron disease)
3) Friedreich's ataxia
4) Syringomyelia, Syringobulbia.
5) Taboparesis (syphilis)
6) Conus medullaris lesion
Dr Khaled Magraby MRCP Notes Neurology 957
Chapter 10: Neurology
Spastic paraparesis
Spastic paraparesis describes an UMN upper motor neuron pattern of weakness in
the lower limbs. (i.e. weakness in both lower limbs, increased tone in both legs and
brisk reflexes with normal upper limbs).
Causes:
1) Cord compression: trauma, tumour
2) Parasagittal meningioma
3) Demyelination e.g. multiple sclerosis
4) Transverse myelitis e.g. HIV
5) Syringomyelia
6) Hereditary spastic paraplegia
7) Tropical spastic paraparesis
8) Osteoarthritis of the cervical spine
Brain lesions (Gross anatomy):
Frontal lobes lesions:
1) Expressive (Broca's) aphasia: located on the posterior aspect of the frontal
lobe, in the inferior frontal gyrus. Speech is non-fluent, laboured, and halting.
(I.e. difficulty in finding the right words whilst speaking).
2) Anosmia.
3) Changes in personality
4) Disinhibition.
5) Primitive reflexes (positive grasp, pout and palmomental reflexes).
6) Urinary and faecal incontinence
7) Perseveration (repeatedly asking same question or doing same task).
8) Inability to generate a list rapidly (For example name animals in 60
seconds or words beginning with the letter F, etc.).
9) Difficulties with executive skills
Frontal lobe dementia is a common neurodegenerative condition. It usually affects
patients of 45-65 years old.
50% of patients presenting with status epilepticus (with no previous history of
seizures) have frontal lobe tumour.
Dr Khaled Magraby MRCP Notes Neurology 958
Chapter 10: Neurology
Parietal lobe lesions:
1) Sensory inattention.
2) Neglect.
3) Apraxias (loss of the ability to execute learned purposeful movements).
4) Astereognosis (tactile agnosia) (inability to recognize object by feeling)
5) Inferior homonymous quadrantanopias
6) Gerstmann's syndrome (lesion of dominant parietal):
I. Alexia (inability to read), قراءة
II. Acalculia (inability to perform mental arithmetic calculation), العد
III. Agraphia (difficulty in writing), كتابة
IV. Dyslexia (inability to recognise letters or words) معرفة الحروف
V. Finger agnosia (difficulty in identifying fingers and naming them)
VI. Right-left disorientation.
Temporal lobe lesion:
1) Wernicke's (recepTive) aphasia: this area 'forms' the speech before
'sending it' to Brocas area. Lesions result in word substitution, neologisms but
speech remains fluent.
2) Superior homonymous quadrantanopias.
3) Auditory agnosia.
4) Prosopagnosia (difficulty recognising faces).
5) Memory impairment.
Occipital lobe lesions:
1) Homonymous hemianopia (with macula sparing)
2) Cortical blindness (blindness due to damage to the visual cortex, may
present as Anton syndrome: there is blindness but the patient is unaware or
denies blindness).
3) Visual agnosia (seeing but not perceiving objects- it is different to neglect
since in agnosia the objects are seen and followed but cannot be named).
Unilateral occipital lobe lesions (left or right) cause contralateral hemianopia or
quadrantanopsia, visual illusions and elementary visual hallucinations.
Dr Khaled Magraby MRCP Notes Neurology 959
Chapter 10: Neurology
Cerebellum lesions:
Midline lesions: gait and truncal ataxia.
Hemisphere lesions: intention tremor, past pointing, dysdiadokinesis,
nystagmus.
More specific areas:
Area Associated conditions
Medial thalamus and Wernicke and Korsakoff syndrome
mammillary bodies of the
hypothalamus
Subthalamic nucleus of the Hemiballism
basal ganglia
Striatum (caudate nucleus) Huntington chorea
of the basal ganglia
Substantia nigra of the Parkinson's disease
basal ganglia
Amygdala Kluver-Bucy syndrome
(hypersexuality, hyperorality,
hyperphagia, visual agnosia
Dorsal column dysfunction: Joint position and light touch.
Spinothalamic dysfunction: Pinprick and temperature.
Thalamic and frontal lobe infarction do not cause visual field defects.
Dr Khaled Magraby MRCP Notes Neurology 960
Chapter 10: Neurology
Visual field defects
Visual field defect is a manifestation of the following pathology:
- Occipital lobe (homonymous hemianopia)
- Temporal lobe (superior quadrantanopia) or
- Parietal lobe (inferior quadrantanopia).
The main points for the exam are:
Right homonymous hemianopia means visual field defect to the right, i.e.
Lesion of left optic tract.
Incongruous defects = optic tract lesion
Congruous defects = optic radiation lesion or occipital cortex
A congruous defect (defect is approximately the same in each eye) simply means
complete or symmetrical visual field loss and conversely an incongruous defect is
incomplete or asymmetric.
Homonymous hemianopia:
Incongruous defects: lesion of optic tract.
Congruous defects: lesion of optic radiation or occipital cortex.
Macula sparing: lesion of occipital cortex.
EX: Left congruous homonymous hemianopia >> lesion in Right occipital cortex.
Homonymous quadrantanopias:
PITS (Parietal-Inferior, Temporal-Superior).
Superior: lesion of temporal lobe.
Inferior: lesion of parietal lobe.
Bitemporal hemianopia:
Lesion of optic chiasm.
Upper quadrant defect > lower quadrant defect = inferior chiasmal
compression, commonly a Pituitary tumour.
Lower quadrant defect > upper quadrant defect = superior chiasmal
compression, commonly a Craniopharyngiomas.
Dr Khaled Magraby MRCP Notes Neurology 961
Chapter 10: Neurology
Nystagmus
It is defined as involuntary oscillations of the eyes.
This may be pendular when the oscillations are equal in rate and amplitude, or
jerking when there are quick and slow phases (the quicker phase is used to define
the direction).
Causes:
1) Visual disturbances.
2) Labyrinth lesions.
3) The central vestibular connections.
4) Brain stem lesion.
5) Cerebellar lesion.
Types:
Nystagmus which changes with the direction of the gaze: involvement of
vestibular nuclei.
Pendular: mostly due to loss of macular vision, but could be in diffuse brain
stem lesions.
Jerking regardless of the direction of gaze: labyrinthine or cerebellar lesion.
Jerking on lateral gaze, and fast in the direction of gaze: brain stem or
cerebellar lesion.
Nystagmus confined to one eye: nerve or muscle lesion, or medial longitudinal
bundle (MLB) lesion.
Dr Khaled Magraby MRCP Notes Neurology 962
Chapter 10: Neurology
Nystagmus restricted to the abducting eye on lateral gaze (ataxic nystagmus):
lesion of the MLB between mid-brain and pons as in multiple sclerosis (MS).
Nystagmus occurring on upward gaze with the fast component upwards
(upbeat nystagmus) may be due to a lesion in the mid-brain at the level of the
superior colliculus.
Downbeat nystagmus (fast phase downwards) suggests a lesion in the lower
part of the medulla. It is therefore typical of the Arnold-Chiari malformation.
Wernicke’s encephalopathy or thiamine deficiency is a rare cause of
downbeat nystagmus.
Upbeat nystagmus: Cerebellar vermis lesions.
Downbeat nystagmus-foramen magnum Arnold-Chiari malformation.
lesions:
MLB-→coordinate lateral rectus of one side with medial rectus of the other side.
Miosis (small pupil): causes:
1) Horner's syndrome
2) Argyll-Robertson pupil عين العاهرة
3) Pontine haemorrhage
4) Congenital
5) Senile miosis
6) Drugs:
Opiates
Organophosphate toxicity
Parasympathomimetics: pilocarpine.
Argyll-Robertson pupil: small irregular pupils that do not react to light but react to
accommodation.
Referred to as the “Whore’s Eye” because of the association with tertiary syphilis
and because of the convenient mnemonic that, like a prostitute, they “can
accommodate but do not react”.
Another mnemonic used for the Argyll-Robertson Pupil (ARP) is Accommodation
Reflex Present (ARP) but Pupillary Reflex Absent (PRA).
Causes: Multiple sclerosis, sarcoidosis, DM.
Dr Khaled Magraby MRCP Notes Neurology 963
Chapter 10: Neurology
Horner's syndrome
Features:
1) Ptosis.
2) Miosis (small pupil).
3) Anhydrosis (loss of sweating one side).
4) Enophthalmos: (sunken eye): (in reality the appearance is due to a narrow
palpebral aperture rather than true Enophthalmos).
Causes: (STC)
Central lesions Pre-ganglionic lesions Post-ganglionic lesions
Anhydrosis of the face, Anhydrosis of the face No anhydrosis
arm and trunk
Stroke Pancoast's tumour Carotid artery dissection
Syringomyelia Thyroidectomy Carotid aneurysm
Multiple sclerosis Trauma Cavernous sinus thrombosis
Tumour Cervical rib Cluster headache
Encephalitis
Horner's syndrome - Anhydrosis determines site of lesion:
Head, arm, trunk = central lesion: stroke, Syringomyelia.
Just face = pre-ganglionic lesion: Pancoast's, cervical rib.
Absent = post-ganglionic lesion: carotid artery.
Distinguishing between causes:
Heterochromia (difference in iris colour) is seen in congenital Horner's
Anhydrosis: see before.
Dr Khaled Magraby MRCP Notes Neurology 964
Chapter 10: Neurology
Ptosis
Ptosis may be unilateral or bilateral.
Causes of bilateral ptosis: Causes of unilateral ptosis,
as bilateral causes plus:
1) Myotonic 1) Horner's
dystrophy. 2) Third nerve palsy
2) Myasthenia gravis
3) Syphilis.
4) Congenital.
Ptosis is much more common in myasthenia gravis than Lambert-Eaton syndrome
Third nerve III palsy
Features:
1) Eye is deviated 'down and out'.
2) Ptosis.
3) Mydriasis (Pupil may be dilated) (sometimes called a 'surgical' third nerve
palsy).
4) Pain only if due to a posterior communicating artery aneurysm.
Ptosis + Miosis (Constricted pupil) >>> Horner's Syndrome
Ptosis + Mydriasis (Dilated pupil) >>> 3 rd. Nerve palsy
Causes:
DM
Vasculitis e.g. temporal arteritis, SLE
False localizing sign due to uncal herniation through tentorium if raised ICP
Posterior communicating artery aneurysm (PCAA) (pupil dilated)
Cavernous sinus thrombosis
Weber's syndrome: Ipsilateral third nerve palsy with contralateral hemiplegia
-caused by midbrain strokes (cerebral peduncle).
Other possible causes: amyloid, multiple sclerosis
Dr Khaled Magraby MRCP Notes Neurology 965
Chapter 10: Neurology
*This term is usually associated with sixth nerve palsies but it may be used for a
variety of neurological presentations
Painful third nerve palsy = posterior communicating artery aneurysm (i.e. EX.
Third nerve palsy with headache and meningism >>> should exclude posterior
communicating artery aneurysm).
Facial nerve
Supply: 'face, ear, taste, and tear’:
1) Face: muscles of facial expression.
2) Ear: nerve to stapedius muscle >> paralysis >> Hyperacusis.
3) Taste: supplies anterior two-thirds of tongue.
4) Tear: parasympathetic fibres to lacrimal glands, also salivary glands >>
Overflow of tears may occur but hyperlacrimation does not.
Causes of unilateral facial nerve palsy:
UMNL LMNL
Stroke - Bell's palsy.
- Ramsay-Hunt syndrome (due to HZV).
- Acoustic neuroma.
- Parotid tumours.
- Multiple sclerosis (may also cause an UMN palsy).
- HIV.
- DM.
Causes of bilateral facial nerve palsy:
1) Sarcoidosis
2) GBS.
3) MG.
4) Polio.
5) Lyme disease.
6) Bilateral Bell’s palsy.
Dr Khaled Magraby MRCP Notes Neurology 966
Chapter 10: Neurology
LMN vs. UMN:
Upper motor neuron lesion 'spares' upper face i.e. forehead
Lower motor neuron lesion affects all facial muscles
Facial palsy + convergent squint→ lesion is in pons as VI th is encircled by VII.
Bell's palsy
Bell's palsy may be defined as an acute, unilateral, idiopathic, facial nerve paralysis.
The aetiology is unknown although the role of the HSV has been investigated
previously.
Features:
Lower motor neuron facial nerve palsy i.e. forehead affected.
Patients may also notice post-auricular pain (may precede paralysis),
altered taste, dry eyes, hyperacusis (present in 30% of cases).
You need to be confident that there are no features of Guillain-Barré (test
reflexes), or brain stem vascular disease or space occupying lesion.
Management:
In the past a variety of treatment options have been proposed including no
treatment, prednisolone only and a combination of acyclovir and prednisolone.
Following a National Institute for Health randomised controlled trial it is now
recommended that prednisolone 25mg bid for 10 days should be
prescribed for patients within 72 hours of onset of Bell's palsy. Adding in
acyclovir gives no additional benefit.
Eye care is important - prescription of artificial tears and eye lubricants should
be considered.
Prognosis:
If untreated around 15% of patients have permanent moderate to severe
weakness.
N.B: Upper motor neuron lesion of facial nerve 'spares' upper face
N.B: A vesicular rash around the ear would suggest a diagnosis of Ramsey Hunt
syndrome.
Dr Khaled Magraby MRCP Notes Neurology 967
Chapter 10: Neurology
Parkinson's disease: features
It is a progressive neurodegenerative condition caused by degeneration of
dopaminergic neurons in the substantia nigra (SN).
Mutations in either the parkin gene or UCHL1 lead to impaired protein
degradation.
This results in a classic triad of features: bradykinesia, tremor and rigidity.
The symptoms of Parkinson's disease are characteristically asymmetrical.
Asymmetrical symptoms (tremors) point towards a diagnosis of idiopathic
Parkinson's disease rather than Parkinsonism of another cause e.g. Drug-
induced Parkinsonism.
Epidemiology:
Around twice as common in men
Mean age of diagnosis is 65 years
1) Bradykinesia:
Poverty of movement also seen, sometimes referred to as hypokinesia.
Short, shuffling steps with reduced arm swinging
Difficulty in initiating movement
2) Tremor:
Typically 'pill-rolling', i.e. in the thumb and index finger
Most marked at rest, with low frequency 3-5 Hz
Worse when stressed or tired and with levodopa
3) Rigidity:
Lead pipe
Cogwheel: due to superimposed tremor
Other characteristic features:
1) Mask-like facies
2) Flexed posture
3) Micrographia
4) Drooling of saliva
Dr Khaled Magraby MRCP Notes Neurology 968
Chapter 10: Neurology
5) Psychiatric features: depression is the most common feature (affects about
40%); dementia, psychosis and sleep disturbances may also occur.
6) Impaired olfaction.
7) REM sleep behaviour disorder.
NB: Diplopia is not common in Parkinson's disease and may suggest an alternative
cause of Parkinsonism such as progressive supranuclear palsy.
NB: Difficulty in initiating movement (bradykinesia), postural instability and unilateral
symptoms (initially) >>> are typical of Parkinson's.
Causes of Parkinsonism:
1) Parkinson's disease
2) Drug-induced e.g. antipsychotics, metoclopramide - see below
3) Progressive supranuclear palsy
4) Multiple system atrophy
5) Wilson's disease
6) Post-encephalitis
7) Dementia pugilistica (secondary to chronic head trauma e.g. boxing)
8) Toxins: carbon monoxide, Carbon disulphide, MPTP (methyl-phenyl
tetrahydropyridine), Manganese and cycad nut
Drug-induced Parkinsonism:
Phenothiazines: e.g. chlorpromazine, prochlorperazine
Butyrophenones: haloperidol, droperidol
Metoclopramide
It has slightly different features to Parkinson's disease:
Motor symptoms are generally rapid onset and bilateral.
Rigidity and rest tremor are uncommon.
N.B: Domperidone does not cross the blood-brain barrier and therefore does not
cause extra-pyramidal side-effects.
NB: Anticholinergic treatment (for example, benzhexol) is the treatment of choice
for tremor predominantly Parkinson's disease.
L-dopa and dopamine agonists are the treatment of choice for bradykinesia and
rigidity.
Dr Khaled Magraby MRCP Notes Neurology 969
Chapter 10: Neurology
Parkinson's disease (PD): management
Currently accepted practice in the management of patients with Parkinson's disease
(PD) is to delay treatment until the onset of disabling symptoms and then to
introduce a dopamine receptor agonist.
1) Dopamine receptor agonists:
Favoured for Pt.>75 Yrs. old.
E.g. Bromocriptine, Cabergoline, Apomorphine, Ropinirole.
Ergot-derived dopamine receptor agonists (Bromocriptine, Cabergoline, and
Pergolide) have been associated with pulmonary, cardiac and
retroperitoneal fibrosis.
Ropinirole is the least associated with tissue fibrosis.
Ropinirole is a non-ergot derived dopamine agonist licensed for ttt of
Parkinsonism & restless leg syndrome.
It is a reasonable choice for first line therapy in younger patients.
The Committee on Safety of Medicines advice that an echocardiogram, ESR,
creatinine and chest x-ray should be obtained prior to treatment and patients
should be closely monitored.
Patients should be warned about the potential for dopamine receptor agonists
to cause impulse control disorders and excessive daytime somnolence.
More likely than levodopa to cause hallucinations in older patients. Nasal
congestion and postural hypotension are also seen in some patients.
Dopamine agonists are thought to be associated with less on/off phenomena
than L-Dopa based therapy and are seen as a reasonable choice for first line
therapy in younger patients.
Pergolide was withdrawn from the US market in March 2007 due to concern
regarding increased incidence of valvular dysfunction
2) Levodopa:
Favoured for Pt. < 75 Yrs. old.
If the patient is elderly, levodopa is sometimes used as an initial treatment.
Usually combined with a decarboxylase inhibitor (e.g. Carbidopa or
benserazide) to prevent peripheral metabolism of levodopa to dopamine.
Dopa-decarboxylase inhibitors reduce the extracerebral complications of L-
dopa therapy. These include nausea, vomiting, postural hypotension and
cardiac arrhythmias.
Dr Khaled Magraby MRCP Notes Neurology 970
Chapter 10: Neurology
Reduced effectiveness with time (usually by 2 years).
Unwanted effects:
1) Dyskinesia (involuntary writhing movements).
2) 'On-off' effect.
3) Psychosis, drowsiness.
4) Postural hypotension
5) Cardiac arrhythmias
6) Dry mouth, anorexia, nausea and vomiting.
7) Reddish discolouration of urine upon standing.
Not use in neuroleptic induced Parkinsonism.
3) MAO-B (Monoamine Oxidase-B) inhibitors:
e.g. Selegiline.
Inhibits the breakdown of dopamine secreted by the dopaminergic neurons.
4) Amantadine:
Mechanism is not fully understood, probably increases dopamine release and
inhibits its uptake at dopaminergic synapses.
SE: include ataxia, slurred speech, confusion, dizziness and livedo reticularis.
5) COMT (Catechol-O-Methyl Transferase) inhibitors:
e.g. Entacapone, tolcapone.
COMT is an enzyme involved in the breakdown of dopamine, and hence may
be used as an adjunct to levodopa therapy.
Used in conjunction with levodopa in patients with established PD.
6) Antimuscarinics:
Block cholinergic receptors.
Now used more to treat drug-induced Parkinsonism (i.e. Parkinsonism
secondary to antipsychotic medications used in ttt of schizophrenia) rather
than idiopathic Parkinson's disease.
Also used in ttt of extrapyramidal symptoms (EPS) (Acute dystonic-dyskinetic
reactions) of metoclopramide antiemetic.
Help tremor and rigidity
E.g. Benzotropine, Procyclidine, Trihexyphenidyl (Benzhexol).
Dr Khaled Magraby MRCP Notes Neurology 971
Chapter 10: Neurology
Progressive Supranuclear Palsy (PSP)
Overview:
It is also known as Steele-Richardson-Olszewski Syndrome (SROS)
A 'Parkinson Plus' syndrome
It is degenerative disease with gradual deterioration and death of selected
areas of the brain. About 6 people per 100,000.
Features: (Triad):
1) Parkinsonism
2) Impairment of vertical gaze due to supranuclear paralysis of upward and
downward gaze (the down gaze worse than up gaze - patients may complain
of difficultly reading or descending stairs)
3) Cognitive impairment
4) Falls
5) Slurring of speech
Management:
Poor response to L-dopa
PSP (Triad): Parkinsonism+ impairment vertical gaze + Cognitive impairment
Multiple system atrophy
Shy-Dragger syndrome is a type of multiple system atrophy.
Features:
1) Parkinsonism
2) Cerebellar signs
3) Autonomic disturbance (disturbance of sphincter control, atonic bladder with
urinary retention , impotence, anhidrosis and postural hypotension)
The Parkinson's symptoms associated with multi-system atrophy respond poorly to
dopamine agonists or L-dopa, in contrast to idiopathic Parkinson's.
Urinary retention can be managed with an indwelling catheter if required,
Postural hypotension managed with support stockings ± mineralocorticoids
Dr Khaled Magraby MRCP Notes Neurology 972
Chapter 10: Neurology
Dementia
Dementia is thought to affect over 700,000 people in the UK and accounts for a large
amount of health and social care spending.
The most common cause of dementia in the UK is Alzheimer's disease followed
by vascular and Lewy body dementia. They may coexist.
Features:
Diagnosis can be difficult and is often delayed.
The Mini-Mental State Examination (MMSE) score is widely used. A score
of ≤ 24 out of 30 suggests dementia.
Management:
1) In primary care a blood screen is usually sent to exclude reversible causes
(e.g. Hypothyroidism).
NICE recommend the following tests: CBC, U&E, LFTs, Ca++, glucose, TFTs,
Vit.B12 and folate levels. Patients are now commonly referred on to old-age
psychiatrists (sometimes working in 'memory clinics').
2) In secondary care neuroimaging is performed* to exclude other reversible
conditions (e.g. Subdural haematoma, normal pressure hydrocephalus) and
help provide information on aetiology to guide prognosis and management.
Neuroimaging is required to diagnose dementia
Alzheimer's disease (AD)
Alzheimer's disease is a progressive degenerative disease of the brain
accounting for the majority of dementia seen in the UK.
Alzheimer's disease is characterised early in the disease by short term memory
loss.
Genetics:
Most cases are sporadic
5% of cases are inherited as an autosomal dominant trait.
Mutations in the amyloid precursor protein (chromosome 21), presenilin 1
(chromosome 14) and presenilin 2 (chromosome 1) genes are thought to
cause the inherited form.
Apoprotein E allele E4 - encodes a cholesterol transport protein.
Dr Khaled Magraby MRCP Notes Neurology 973
Chapter 10: Neurology
Pathological changes:
1) Macroscopic = widespread cerebral atrophy, particularly involving the cortex
and hippocampus.
2) Microscopic = intraneuronal neurofibrillary tangles, neuronal plaques,
deficiency of neurons.
3) Biochemical = deposition of type A-Beta-amyloid protein in cortex, deficit of
Ach from damage to an ascending forebrain projection.
Neurofibrillary tangles:
Paired helical filaments are partly made from a protein called tau
In AD are tau proteins are excessively phosphorylated
Management:
1) NICE now recommend the three acetyl cholinesterase inhibitors (↑Ach):
(Donepezil (Aricept®), Rivastigmine (Exelon®) and (Galantamine) as
options for managing mild to moderate Alzheimer's disease.
2) Memantine (Namenda®): (a NMDA receptor antagonist) is reserved for
patients with moderate - severe Alzheimer's.
Donepezil → SE: Insomnia, Bradycardia, Heart block and UB obstruction
The NICE guidelines recommend discontinuation of cholinesterase inhibitors as
Donepezil once the mini mental state examination has fallen below 12/30 and
possibly consider Memantine, which is licensed for use in moderate to severe
dementia.
So pt. with Dementia on Donepezil (Aricept®), then ↓ MMSE >>> so withdraw
Donepezil and consider Memantine.
Lewy body dementia (LBD)
Lewy body dementia is an increasingly recognised cause of dementia, accounting for
up to 20% of cases.
It is a mixture of Alzheimer's disease with Parkinson's disease.
The characteristic pathological feature is alpha-synuclein cytoplasmic inclusions
(Lewy bodies) in the substantia nigra, paralimbic and neocortical areas.
The relationship between Parkinson's disease and Lewy body dementia is
complicated, particularly as dementia is often seen in Parkinson's disease.
Dr Khaled Magraby MRCP Notes Neurology 974
Chapter 10: Neurology
Also, up to 40% of patients with Alzheimer's have Lewy bodies.
Neuroleptics should be avoided in Lewy body dementia as patients are extremely
sensitive and may develop irreversible Parkinsonism.
Questions may give a history of a patient who has deteriorated following the
introduction of an antipsychotic agent (like haloperidol).
EX: A 64-year-old man who is under investigation for parkinsonian symptoms is
brought to the GP by his wife. She is concerned her husband is becoming
increasingly agitated. The GP prescribes haloperidol. One week later the
parkinsonian symptoms have deteriorated markedly. What is the most likely
underlying diagnosis? >>> Lewy body dementia.
Features:
1) Progressive cognitive impairment.
2) Parkinsonism.
3) Hallucinations (visual or non-visual hallucinations, other features such as
delusions may also be seen).
4) Symptoms worsen with neuroleptics/ antipsychotic agent.
Diagnosis:
Usually clinical.
Single-photon emission computed tomography (SPECT) is increasingly used.
It is currently commercially known as a DaTscan (dopamine transporter
scan).
o Dopaminergic iodine-123-radiolabelled 2-carbomethoxy-3-(4-
iodophenyl)-N-(3-fluoropropyl) nortropane (123-I FP-CIT) is used as
the radioisotope.
o The sensitivity of SPECT in diagnosing Lewy body dementia is around
90% with a specificity of 100%.
o Its main drawback is expense.
The findings on conventional imaging such as MRI are generally non-specific.
Lewy body dementia has no specific identifying features on CT or MRI.
Normal pressure hydrocephalus
It is a reversible cause of dementia seen in elderly patients.
It is thought to be secondary to reduced CSF absorption at the arachnoid villi.
Dr Khaled Magraby MRCP Notes Neurology 975
Chapter 10: Neurology
These changes may be secondary to head injury, subarachnoid haemorrhage or
meningitis.
A classical triad of features is seen:
1) Dementia and bradyphrenia
2) Urinary incontinence
3) Gait abnormality (may be similar to Parkinson's disease)
Urinary incontinence + gait abnormality + dementia = normal pressure hydrocephalus.
Imaging: Hydrocephalus with an enlarged fourth ventricle.
Management: VPS (Ventriculo-peritoneal shunting).
Creutzfeldt –Jakob Disease (CJD)
Rapidly progressive, severe invariably fatal usually within few months.
Dementia
Cerebellar ataxia.
Diffuse myoclonic jerks: it is typical and progressive, even during the later
stage when the patient is stuporous or comatose.
Ataxia and involuntary movements (for example, myoclonus) usually appear
at an interval of about 6 months after the initial symptoms.
In the majority of the cases the first symptoms are psychiatric (depression,
irritability) and painful sensory symptoms in the LLs.
New variant CJD usually presents in a young person, in their twenties or
thirties.
EEG is usually normal in new variant CJD.
Rapid cognitive decline in a young person with myoclonus is strongly
suggestive of Creutzfeldt-Jakob disease (CJD).
Investigations:
EEG: characteristic diffuse non-specific slowing periodic high amplitude
sharp wave complexes PSWCs of 1-2 Hz, but diagnosis relies on either
specialized tests for prion protein in CSF or brain biopsy.
MRI: Pulvinar sign > 90% (bilateral posterior thalamic nuclei high signal
abnormalities) pathological in variant CJD (not sporadic).
CSF:14-3-3 protein
Prion protein in tonsils.
Sporadic CJD:
It is a prion disease.
Dr Khaled Magraby MRCP Notes Neurology 976
Chapter 10: Neurology
Normal prion proteins are found on cell surfaces all over the body mainly in an
alpha helix structure.
Predominantly affects late mid aged individuals with mean age of death in the
late 60s.
Memory impairment and subsequently rapidly progressive dementia from
few weeks duration to less than 2 years.
Commonly accompanying features include cerebellar ataxia, pyramidal and
extrapyramidal signs and myoclonus which are common early features.
A definitive diagnosis can only be made post-mortem.
The median duration of illness is 4 months and about 65% of cases die within
6 months.
EX: Male pt. 60 years with increasing forgetfulness over the last 3 months, he
become more agitated and police brought him home after seeing him wondering in
the street, O/E: he has broad based gait, intention tremor, past pointing, myoclonic
jerk, brisk reflexes and bilateral up-going plantars >>> Sporadic CJD.
Meningitis: CSF analysis
Normal CSF:
Opening pressure 5-18 cm H2O (60-150 mm)
Glucose 3.3-4.4 mmol (> 2/3 blood glucose)
Protein 0.15-0.45 g/l
WBC ˂ 5/ mm³
The characteristic cerebrospinal fluid (CSF) findings in meningitis:
Bacterial Viral Tuberculous
Appearance Cloudy Clear/cloudy Fibrin web
Glucose Low (< 1/2 Normal* Low (< 1/2 plasma)
plasma)
Protein High (> 1 g/l) Normal/raised High (> 1 g/l)
White cells 10 - 5,000 15 - 1,000 10 - 1,000
polymorphs/mm³ lymphocytes/mm³ lymphocytes/mm³
Dr Khaled Magraby MRCP Notes Neurology 977
Chapter 10: Neurology
The following conditions are associated with raised lymphocytes:
1) Viral meningitis/encephalitis
2) TB meningitis
3) Partially treated bacterial meningitis
4) SLE
5) Behcet's
6) Leukaemia
7) Lymphoma
8) Lyme disease
The following conditions are associated with raised protein levels:
1) Bacterial, Tuberculous, Fungal meningitis
2) Viral encephalitis
3) GBS (Guillain-Barre syndrome)
4) Spinal block (Froin's syndrome): It describes an increase in CSF protein
below a spinal canal blockage (e.g. tumour, disc, infection).
The Ziehl-Neelsen stain is only 20% sensitive in the detection of tuberculous
meningitis and therefore PCR is sometimes used (sensitivity = 75%).
*Mumps is unusual in being associated with a low glucose level in a proportion of
cases. A low glucose may also be seen in herpes encephalitis
Post-lumbar puncture headache
Headache following lumbar puncture (LP) occurs in approximately one-third of
patients. The pathophysiology of is unclear but may relate to a 'leak' of CSF following
dural puncture.
It is more common in young females with a low body mass index.
Typical features:
1) Usually develops within 24-48 hours following LP but may occur up to one
week later.
2) May last several days.
3) Worsens with upright position.
4) Improves with recumbent position.
Dr Khaled Magraby MRCP Notes Neurology 978
Chapter 10: Neurology
Factors which may contribute to Factors which do not contribute to
↑↑↑headache headache
Increased needle size Increased volume of CSF removed
Direction of bevel Bed rest following procedure
Not replacing the stylet Increased fluid intake post procedure
Increased number of LP attempts Opening pressure of CSF
Position of patient
Management:
Supportive initially (analgesia, rest).
If pain continues for more than 72 hours then specific treatment is indicated,
to prevent subdural haematoma.
TTT options include: blood patch, epidural saline and caffeine I.V.
Herpes simplex encephalitis (HSE)
Herpes simplex (HSV) encephalitis is a very common topic in the exam.
The virus characteristically affects the temporal lobes - questions may give the
result of imaging or describe temporal lobe signs e.g. aphasia.
Features:
Fever, headache, vomiting.
Psychiatric symptoms, change in personality, odd behaviour, becoming
aggressive over trivial things, confusion, seizures.
Focal features e.g. aphasia.
Peripheral lesions (e.g. cold sores) have no relation to presence of HSV
encephalitis.
Pathophysiology:
HSV-1 responsible for 95% of cases in adults
Typically affects temporal and inferior frontal lobes
Investigations:
CSF: lymphocytosis, elevated protein
PCR for HSV (the quickest method).
Dr Khaled Magraby MRCP Notes Neurology 979
Chapter 10: Neurology
CT/ MRI (MRI is better): medial temporal and inferior frontal changes (e.g.
petechial haemorrhages), low attenuation areas with surrounding oedema -
normal in one-third of patients.
EEG: lateralised periodic discharges at 2 Hz (BUT is not diagnostic).
TTT: I.V Acyclovir (10-15 mg/kg IV every 8 hrs for 10-14 days).
The prognosis is dependent on whether acyclovir is commenced early. If
treatment is started promptly the mortality is 10-20%.
Left untreated the mortality approaches 80%.
EX: A 34-year-old woman who presents with confusion, headache and fever is
admitted to the ER. Shortly after admission she has a seizure. A MRI scan is
performed which shows patchy haemorrhagic changes in the temporal lobe. Given
the likely diagnosis, what is the treatment of choice? I.V Acyclovir
HIV: Neurocomplications
HIV Neuro complications
Generalised Focal
1) Encephalitis 1) Toxoplasmosis
2) Cryptococcus 2) Lymphoma
3) PML 3) TB
4) AIDS dementia complex
Generalised neurological disease:
1) Encephalitis
May be due to CMV or HIV itself
HSV encephalitis but is relatively rare in the context of HIV
CT: oedematous brain
2) Cryptococcus:
Most common fungal infection of CNS: Cryptococcus neoformans.
Headache, fever, malaise, nausea/vomiting, seizures, focal neurological
deficit.
Dr Khaled Magraby MRCP Notes Neurology 980
Chapter 10: Neurology
Meningitis is typical presentation but may occasionally cause a space
occupying lesion
CSF: high opening pressure, India ink test positive.
CT: meningeal enhancement, cerebral oedema
3) Progressive multifocal leukoencephalopathy (PML):
Widespread demyelination.
Due to infection of oligodendrocytes by human papovirus JC virus (a
polyoma DNA virus).
Symptoms, subacute onset: behavioural changes, speech, motor, visual
impairment, ataxia, Head tremor, Focal neurology progressing over a period
of months to paresis and even coma.
Diagnosed by CSF PCR for the JC virus.
CT: single or multiple lesions, no mass effect, don't usually enhance. Several
areas of Low attenuation diffusely.
MRI is better - high-signal demyelinating white matter lesions are seen.
4) AIDS dementia complex:
Caused by HIV virus itself
Symptoms: behavioural changes, motor impairment
CT: cortical and subcortical atrophy
Focal neurological lesions:
1) Toxoplasmosis:
Accounts for around 50% of cerebral lesions in patients with HIV.
Constitutional symptoms, headache, confusion, drowsiness
CT: usually single or multiple ring enhancing lesions, mass effect may be
seen
Management: sulfadiazine and pyrimethamine
2) Primary CNS lymphoma:
Accounts for around 30% of cerebral lesions
Associated with the Epstein-Barr virus
CT: single or multiple homogenous enhancing lesions
Dr Khaled Magraby MRCP Notes Neurology 981
Chapter 10: Neurology
Treatment generally involves steroids (may significantly reduce tumour size),
chemotherapy (e.g. methotrexate) + with or without whole brain irradiation.
Surgical may be considered for lower grade tumours.
Differentiating between toxoplasmosis and lymphoma is a common clinical scenario
in HIV patients.
It is clearly important given the vastly different treatment strategies.
Toxoplasmosis Lymphoma
Multiple lesions Single lesion
Ring or nodular enhancement Solid (homogenous) enhancement
Thallium SPECT negative Thallium SPECT positive
3) Tuberculosis:
Much less common than toxoplasmosis or primary CNS lymphoma.
CT: single enhancing lesion.
Head injury: NICE guidance
NICE has strict and clear guidance regarding which adult patients are safe to
discharge and which need further CT head imaging.
The former group are also divided into two further cohorts, those who require an
immediate CT head and those requiring CT head within 8 hours of injury:
CT head immediately:
1) GCS < 13 on initial assessment.
2) GCS < 15 at 2 hours post-injury.
3) Suspected open or depressed skull fracture.
4) Any sign of basal skull fracture (haemotympanum, 'panda' eyes, and CSF
leakage from the ear or nose, Battle's sign).
5) Post-traumatic seizure.
6) Focal neurological deficit.
7) More than 1 episode of vomiting
Dr Khaled Magraby MRCP Notes Neurology 982
Chapter 10: Neurology
CT head scan within 8 hours of the head injury:
For adults with any of the following risk factors who have experienced some loss of
consciousness or amnesia since the injury:
1) Age 65 years or older.
2) Any history of bleeding or clotting disorders.
3) Dangerous mechanism of injury (a pedestrian or cyclist struck by a motor
vehicle, an occupant ejected from a motor vehicle or a fall from a height of
greater than 1 metre or 5 stairs).
4) More than 30 minutes' retrograde amnesia of events immediately before the
head injury.
N.B: If a patient is on warfarin who have sustained a head injury with no other
indications for a CT head, perform a CT head scan within 8 hours of the injury.
Head injury: types of traumatic brain injury
Basics:
Primary brain injury may be focal (contusion/haematoma) or diffuse (diffuse
axonal injury).
Diffuse axonal injury occurs as a result of mechanical shearing following
deceleration, causing disruption and tearing of axons.
Intra-cranial haematomas can be extradural, subdural or intra-cerebral, while
contusions may occur adjacent to (coup) or contralateral (contre-coup) to the
side of impact.
Secondary brain injury occurs when cerebral oedema, ischaemia, infection,
tonsillar or tentorial herniation exacerbates the original injury.
The normal cerebral auto regulatory processes are disrupted following
trauma rendering the brain more susceptible to blood flow changes and
hypoxia.
The Cushings reflex (hypertension and bradycardia) often occurs late and
is usually a pre terminal event.
Dr Khaled Magraby MRCP Notes Neurology 983
Chapter 10: Neurology
Type of injury Notes
Extradural Bleeding into the space between the dura mater and the skull.
(epidural)
haematoma Often results from acceleration-deceleration trauma or a blow to
the side of the head.
The majority of extradural haematomas occur in the temporal
region where skull fractures cause a rupture of the middle
meningeal artery.
Features:
Features of raised intracranial pressure.
Some patients may exhibit a lucid interval.
A history of a closed head injury, followed by a lucid period
followed by decreasing level of consciousness.
Subdural Bleeding into the outermost meningeal layer.
haematoma
Most commonly occur around the frontal and parietal lobes.
(SDH)
Risk factors include old age, alcoholism and anticoagulation.
Slower onset of symptoms than an epidural haematoma.
Fluctuating confusion/consciousness >>> SDH.
Subarachnoid Usually occurs spontaneously in the context of a ruptured
haemorrhage cerebral aneurysm but may be seen in association with other
injuries when a patient has sustained a traumatic brain injury.
(SAH)
EX: A 14-year-old boy bangs heads with an opponent during an unofficial football
game. He is knocked out for 30 seconds and is amnesic for the event. He recovers
quickly and continues playing.
Two hours later he complains of headache, begins vomiting then loses
consciousness >>> Extra-Dural haematoma >>> CT brain.
Dr Khaled Magraby MRCP Notes Neurology 984
Chapter 10: Neurology
Subdural haemorrhage (SDH)
Basics:
Most commonly secondary to trauma e.g. old person/alcohol falling over.
Initial injury may be minor and is often forgotten.
Caused by bleeding from damaged bridging veins between cortex and venous
sinuses.
Features:
Classically fluctuating conscious level (episodes of confusion and between
these episodes the pt. is apparently his/her normal self).
Old patient with Fluctuating consciousness = subdural haemorrhage (SDH)
Treatment: Neurosurgical review >>>? Burr hole.
Subarachnoid haemorrhage (SAH)
Causes:
1) 85% are due to rupture of berry aneurysms: (conditions associated with
berry aneurysms include adult polycystic kidney disease, Ehlers-Danlos
syndrome and coarctation of the aorta).
2) AV malformations.
3) Trauma.
4) Tumours.
Investigations:
1) CT: negative in 5%.
2) LP: done after 12 hrs (for allowing time for xanthochromia to develop).
3) Cerebral angiography.
Posterior communicating artery aneurysms can compress the third cranial
nerve. If the aneurysm ruptures it can cause the classic picture of an ipsilateral
painful third nerve palsy, with the eye down and out, ptosis, and pupil dilation.
Anterior communicating artery aneurysms do not compress the third nerve.
Dr Khaled Magraby MRCP Notes Neurology 985
Chapter 10: Neurology
Complications:
1) Rebleeding (in 30%).
2) Obstructive hydrocephalus (due to blood in ventricles).
3) Vasospasm leading to cerebral ischaemia.
Management:
1) Nimodipine (e.g. 60mg / 4 hr, if BP allows) has been shown to reduce the
severity of neurological deficits but doesn't reduce rebleeding. (The way
Nimodipine works in SAH is not fully understood. It has been previously
postulated that it reduces cerebral vasospasm hence maintaining cerebral
perfusion but this has not been demonstrated in studies).
2) Neurosurgical opinion: no clear evidence over early surgical intervention
against delayed intervention.
N.B: Intra-cranial haemorrhage can cause changes in the ECG which are typically
deep symmetrical T-wave inversion and prolonged QT interval.
Acute confusional state (Delirium)
Acute confusional state is also known as delirium or acute organic brain syndrome.
It affects up to 30% of elderly patients admitted to hospital.
Features - wide variety of presentations:
Memory disturbances (loss of short term > long term)
May be very agitated or withdrawn
Disorientation
Mood change
Visual hallucinations
Disturbed sleep cycle
Poor attention
Management:
Treatment of underlying cause
Modification of environment
Dr Khaled Magraby MRCP Notes Neurology 986
Chapter 10: Neurology
The 2006 Royal College of Physicians publication 'the prevention, diagnosis
and management of delirium in older people: concise guidelines'
recommended haloperidol 0.5 mg as the first-line sedative.
The 2010 NICE delirium guidelines advocate the use of haloperidol or
olanzapine.
N.B: Whilst many doctors may use oral lorazepam in this situation the Royal College
of Physicians recommend haloperidol as the first-line sedative.
Myasthenia gravis (MG)
Myasthenia gravis is an acquired humoral autoimmune disorder resulting in
insufficient functioning acetylcholine receptors.
Acetylcholine Receptors Auto Antibodies (ACHRAB) are seen in 90% of cases
(antibodies are less commonly seen in disease limited to the ocular muscles). These
antibodies block Ach receptors at the post-synaptic neuromuscular junction.
It is well known to be associated with other autoimmune diseases such as
pernicious anaemia, thyroid disease and rheumatoid arthritis.
Myasthenia is more common in women (2:1).
It is a neuromuscular disease leading to fluctuating muscle weakness & fatigability.
The key feature is muscle fatigability - muscles become progressively weaker
during periods of activity later in the day after prolonged use of specific muscle and
slowly improve after periods of rest:
1) Proximal muscle weakness: face, neck, limb girdle.
2) Extra ocular muscle weakness: diplopia and Ptosis.
3) Dysphagia (worse with liquids than solids).
4) Dysarthria, slurred speech.
Associations:
1) Thymomas in 15%
2) Thymic hyperplasia in 50-70%
3) Autoimmune disorders: Autoimmune thyroid disorders, RA, SLE, pernicious
anaemia.
30-40% of Thymoma cases have coexistent myasthenia gravis.
Thymoma has equal sex distribution and rarely presents ˂ age of 20 years.
Dr Khaled Magraby MRCP Notes Neurology 987
Chapter 10: Neurology
Exacerbating factors:
The most common exacerbating factor is exertion resulting in fatigability, which is the
hallmark feature of MG. Symptoms become more marked during the day.
The following drugs may exacerbate myasthenia:
1) Penicillamine
2) Procainamide
3) Quinidine
4) Beta-blockers
5) Lithium
6) Phenytoin
7) Gentamicin
Investigations:
1) Tensilon test: IV Edrophonium reduces muscle weakness temporarily and
is sometimes used in the diagnosis of myasthenia gravis (Diagnostic and
therapeutic test).
2) Single fibre electromyography (EMG) is the most sensitive test for
myasthenia gravis.
3) CT thorax to exclude thymoma.
4) CPK normal.
5) Muscle fatigability can be demonstrated at the bedside by asking the patient
to count aloud from 1 to 20 slowly - often inducing slurred speech.
6) Fatigable ptosis can be demonstrated by asking the patient to maintain
upward gaze without blinking for 30-60 seconds.
Management:
1) Long-acting anticholinesterase e.g. Pyridostigmine.
2) Immunosuppression: prednisolone initially.
3) Thymectomy: is significantly improve symptoms of myasthenia, and is the
next logical step once diagnosis of myasthenia is confirmed.
Management of myasthenic crisis:
1) IVIG (Intravenous immunoglobulins)
2) Plasmapharesis
Dr Khaled Magraby MRCP Notes Neurology 988
Chapter 10: Neurology
Opinions vary as to whether Plasmapharesis or IVIG should be given as a first-line.
Plasmapharesis usually works quicker but involves more expensive equipment.
Mechanical dysphagia (for example, oesophageal and gastric carcinoma,
oesophageal stricture, etc.) causes dysphagia that is worse with solids than liquids.
Nasal regurgitation and dysarthria are not usually accompanying features of
mechanical dysphagia.
Neurogenic dysphagia that is worse with liquids than solids also associated with
dysarthria, nasal regurgitation, coughing and choking episodes during meals.
Achalasia typically affects solids more than liquids, or both solids and liquids
equally. It typically presents at earlier age (25 to 40-years-old). Chest pain is a
predominant feature, with no weight loss.
NB: Ocular myasthenia: bilateral ptosis and variable diplopia, without proptosis
or injection of the eyes, with normal all pupils reflexes.
Lambert-Eaton Myasthenic Syndrome (LEMS)
It is seen in 50 % of cases associated with small cell lung cancer (SCLC), and to a
lesser extent breast and ovarian cancer.
It is a rare paraneoplastic phenomenon which can precede radiological evidence of
the tumour by up to 5 years.
It may also occur independently as an autoimmune disorder.
It is caused by an antibody directed against pre-synaptic voltage gated calcium
channel (VGCC) in the peripheral nervous system leading to failure of acetyl
choline release.
Features:
1) Repeated muscle contractions after a few minutes will lead to increased
muscle strength and return of absent reflexes (in contrast to myasthenia
gravis). (In reality this is seen in only 50% of patients and following prolonged
muscle use muscle strength will eventually decrease).
2) Limb girdle (proximal muscle) weakness (affects lower limbs first).
3) Hypo-reflexia to areflexia (but normalise with repetitive muscle contraction).
4) Autonomic symptoms: dry mouth, impotence, difficultly micturating.
5) No bulbar or extraocular muscle involvement like ophthalmoplegia and ptosis
(unlike in myasthenia gravis).
Dr Khaled Magraby MRCP Notes Neurology 989
Chapter 10: Neurology
6) No wasting or fasciculations.
7) No Sensory affection.
Diagnosis:
- EMG: Incremental response to repetitive electrical stimulation.
- Anti-VGCC auto Antibodies: are found in 95% of cases.
Management:
1) Treatment of underlying cancer.
2) Immunosuppression, for example with prednisolone ± azathioprine.
3) 3,4-diaminopyridine IV is currently being trialled (works by blocking
potassium channel efflux in the nerve terminal so that the action potential
duration is increased. Calcium channels can then be open for a longer time
and allow greater acetylcholine release to the stimulate muscle at the end
plate).
4) IVIG therapy and plasma exchange may be beneficial.
Comparison between Myasthenia gravis (MG) & Lambert-Eaton Myasthenic
Syndrome (LEMS):
MG LEMS
Muscle strength ↓ during ongoing Delayed maximum
exercise contraction
Ocular muscle affection Yes NO
ANS affection NO Yes
Tendon reflexes Normal ↓ with post-tetanic
facilitation
Single nerve stimulation Normal Reduced
(Amplitude)
Repetitive stimulation ↓ at 3-Hz stimulation Additional ↑ at 20-Hz
stimulation
Ach Receptor Ab Positive -----
VGCC Ab ------- Positive
Dr Khaled Magraby MRCP Notes Neurology 990
Chapter 10: Neurology
Eaton (1905-1958) U.S. neurologist at Mayo Clinic. Lambert (1915-2003) U.S
neuro-physiologist at Mayo Clinic and Prof. of physiology at University of Minnesota.
Narcolepsy
It is excessive daytime somnolence and an overwhelming desire to sleep.
Symptoms include excessive daytime sleepiness (EDS), involuntary sleep episodes,
catalepsy (70%), sleep paralysis hallucinations – hypnagogic (at the onset of sleep)
and hypnopompic (on awaking).
Cataplexy
Cataplexy describes the sudden and transient loss of muscular tone caused by
strong emotion (e.g. laughter, being frightened).
Around two-thirds of patients with narcolepsy have cataplexy.
Features range from buckling knees to collapse.
Neuroleptic malignant syndrome (NMS)
It is a rare but dangerous condition seen in patients taking antipsychotic
medication (e.g. Olanzapine haloperidol, chlorpromazine and Citalopram).
It carries a mortality of up to 10% and can also occur with atypical antipsychotics.
It may also occur with dopaminergic drugs (such as levodopa) for Parkinson's
disease, usually when the drug is suddenly stopped or the dose reduced.
It is thought to arise from blockage of dopamine receptors or decrease in availability
of dopamine (↓ Dopamine).
Primary diagnostic features are altered conscious level, autonomic instability and
muscular rigidity with raised CPK
The syndrome can occur within hours of initiating drug therapy, but typically takes
about 1 week.
The mortality is between 10-20%.
Features:
More common in young male patients
Onset usually in first 10 days of treatment or after increasing dose of
antipsychotic.
Dr Khaled Magraby MRCP Notes Neurology 991
Chapter 10: Neurology
Pyrexia
Altered mental status
Muscle rigidity (sometimes with dysphagia and dysarthria).
Extrapyramidal signs, tremors, catatonia, muteness.
Tachycardia
Autonomic dysfunction.
A raised CPK & AST is present in most cases.
A leucocytosis may also be seen.
Renal failure may occur secondary to rhabdomyolysis
Metabolic acidosis.
DIC.
Management:
Stop antipsychotic.
IV fluids to prevent renal failure.
Dopamine agonist: Bromocriptine or Levodopa.
Dantrolene IV was formerly recommended as monotherapy, case series
suggests that when it is used in the absence of Bromocriptine, mortality may
actually be increased.
Dantrolene is thought to work by decreasing excitation-contraction coupling in
skeletal muscle by binding to the ryanodine receptor, and decreasing the release of
calcium from the sarcoplasmic reticulum.
EX: A 35-year-old man with a history of schizophrenia is brought to the ED by
worried friends due to drowsiness. On examination he is generally rigid. >>> A
diagnosis of neuroleptic malignant syndrome should be suspected.
Epilepsy: classification
Basics:
Two main categories are generalised and partial seizures
Partial seizures may progress to general seizures
Other types: myoclonic, atypical absence, atonic and tonic seizures are
usually seen in childhood
Dr Khaled Magraby MRCP Notes Neurology 992
Chapter 10: Neurology
Generalised - no focal features, consciousness lost immediately:
1) Grand mal (tonic-clonic)
2) Petit mal (absence seizures)
3) Myoclonic jerks: brief, rapid muscle jerks
4) Partial seizures progressing to generalised seizures
5) Infantile spasms
Partial - focal features depending on location:
1) Simple (no disturbance of consciousness or awareness)
2) Complex (consciousness is disturbed)
3) Simple partial followed by impaired consciousness, or consciousness
impaired at onset
4) Partial seizures with secondary generalisation
5) Temporal lobe → aura, déjà vu, jamais vu; motor → Jacksonian
With simple partial seizures there is no disturbance of consciousness or awareness.
Lip smacking is an example of an automatism- an automatic, repetitive act.
Absence seizures (petit mal)
It is a form of generalised epilepsy that is mostly seen in children.
The typical age of onset of 3-10 years old and girls are affected twice as commonly
as boys.
Features: طفلة بتسرح كذا مرة فى اليوم لثوانى وترجع تانى طبيعية وال تتذكر ذلك
1) Absences last a few seconds and are associated with a quick recovery.
2) Seizures may be provoked by hyperventilation or stress.
3) The child is usually unaware of the seizure.
4) They may occur many times a day.
5) EEG: bilateral, symmetrical 3Hz spike and wave pattern.
Management:
Sodium valproate and ethosuximide are first-line treatment
Good prognosis - 90-95% become seizure free in adolescence.
Dr Khaled Magraby MRCP Notes Neurology 993
Chapter 10: Neurology
N.B: Carbamazepine may actually exacerbate absence seizure.
NB: Gelastic seizures should be suspected in cases of erratic laughing or crying.
It can be hard to identify in young children but there is usually associated
automatisms such as fidgeting or lip smacking or change in sensorium.
Myoclonic epilepsy:
Juvenile myoclonic epilepsy is the commonest of the idiopathic generalised
epilepsies.
Precipitating factors include alcohol, menstruation and sleep deprivation.
The condition is genetically linked to the short arm of chromosome 6.
Prognosis is extremely favourable if the condition is treated correctly, with many
patients becoming seizure-free.
TTT options include: sodium valproate, lamotrigine and topiramate.
Lifelong drug treatment is usually necessary to avoid relapses in patients who
achieve seizure-free status on medication.
Epilepsy: treatment
Most neurologists now start antiepileptic drugs (AED) following a second epileptic
seizure.
NICE guidelines suggest starting antiepileptic after the first seizure if any of the
following are present: (4):
1) The patient has a neurological deficit.
2) Brain imaging shows a structural abnormality.
3) EEG shows unequivocal epileptic activity.
4) The patient or their family consider the risk of having a further seizure
unacceptable.
Monotherapy should be attempted before combination therapy is started.
A study of patients with previously untreated epilepsy demonstrated that 47%
achieved control of seizures with the use of their first single drug.
14% became seizure-free during treatment with a second or third drug.
An additional 3% became seizure-free with the use of two drugs simultaneously.
Dr Khaled Magraby MRCP Notes Neurology 994
Chapter 10: Neurology
Sodium valproate is considered the first line treatment for patients with
generalised seizures with carbamazepine used for partial seizures.
Epilepsy medication: first-line:
Generalised seizure: Sodium valproate
Partial seizure: Carbamazepine
Generalised tonic-clonic seizures:
1) Sodium valproate
2) Second line: Lamotrigine (Lamictal®), Carbamazepine
Absence seizures (Petit mal):
Sodium valproate or Ethosuximide
Sodium valproate particularly effective if co-existent tonic-clonic seizures in
primary generalised epilepsy.
Myoclonic seizures:
1) Sodium valproate
2) Second line: Lamotrigine, clonazepam
Partial seizures:
Carbamazepine
Second line: lamotrigine**, sodium valproate
Gabapentin (Neurontin®)
Phenytoin (Epanutin ®)
Phenytoin is used to in the management of seizures.
Mechanism of action:
Na channel blocker, decreasing the sodium influx into neurons which in turn
decreases excitability.
Dr Khaled Magraby MRCP Notes Neurology 995
Chapter 10: Neurology
Adverse effects:
Phenytoin is associated with a large number of adverse effects.
These may be divided into acute, chronic, idiosyncratic and teratogenic.
1) Acute:
Initially: Cerebellum syndrome: Ataxia, Dizziness, Diplopia, Nystagmus,
Chorea, Slurred speech.
Later: confusion, seizures.
Phenytoin toxicity typically gives rise to a cerebellar-like syndrome
2) Chronic:
Common: gingival hyperplasia (secondary to increased expression of
platelet derived growth factor, PDGF), hirsutism )(شعر, acne, coarsening of
facial features, drowsiness.
Megaloblastic anaemia (secondary to altered Folate metabolism).
Pancytopenia
Peripheral neuropathy.
Enhanced vitamin D metabolism causing osteomalacia.
Lymphadenopathy, Pseudolymphoma or, rarely, malignant lymphoma
and mycosis-fungoides-like lesions.
R/ Epanutin + Folic acid + Vit B complex + Pregabalin+ Calcium with Vit D
3) Idiosyncratic:
Fever
Rashes, up to severe reactions as toxic epidermal necrolysis (TEN)
Hepatitis
Dupuytren's contracture (although not listed in the BNF).
Aplastic anaemia
Drug-induced lupus
4) Teratogenic:
Associated with cleft palate and congenital heart disease.
Dr Khaled Magraby MRCP Notes Neurology 996
Chapter 10: Neurology
Usage of phenytoin in renal impairment:
Patient with renal failure, a state in which drugs that are usually highly protein
bound, such as phenytoin, sodium valproate and warfarin. Lose some of their
affinity for protein binding. This results in increased availability of free drug at
any given dose, which then increases the risk of toxicity.
So you will find signs of phenytoin toxicity but with normal Phenytoin levels in
blood by the lab
Because laboratory assays for phenytoin usually measure total drug concentration,
this gives a degree of false reassurance.
So in patients with renal failure, dose reduction of phenytoin is required.
Sodium valproate (Depakin®):
It is used in the management of epilepsy and is first line therapy for generalised
seizures. It works by increasing ↑GABA activity.
Adverse effects:
1) GIT: anorexia, nausea, vomiting
2) PCOS like picture: weight gain, acne, hair loss: so ↓ use in females.
3) Highly teratogenic: neural tube defects.
4) Alopecia ( صلعin 12 % of cases): regrowth may be curly
5) Ataxia
6) Tremor
7) Hepatitis
8) Pancreatitis
9) Thrombocytopenia
10) Hyponatraemia
11) Enzyme Inhibitor P450 enzyme system
Na valproate SE: nausea, ↑ appetite, ↑ weight gain, Alopecia )(أصلع, Ataxia,
Hepatitis, Pancreatitis and Teratogenic.
Sodium valproate can occasionally have an idiosyncratic response leading to
severe or even fatal hepatic toxicity.
Dr Khaled Magraby MRCP Notes Neurology 997
Chapter 10: Neurology
Vigabatrin (Sabril 500 mg tab):
40% of patients develop visual field defects (constriction of visual fields),
which may be irreversible.
Visual fields should be checked every 6 months.
Other side effect: alopecia.
N.B: The 2007 SANAD study indicated that lamotrigine may be a more suitable first-
line drug for partial seizures although this has yet to work its way through to
guidelines.
NB: Lamotrigine is associated with skin rash (and Stevens-Johnson syndrome in
severe cases).
N.B: Monotherapy with another drug should be attempted before combination
therapy is started. Caution should be exercised when combining sodium valproate
and lamotrigine as serious skin rashes such as Steven-Johnson's syndrome may
be provoked.
N.B: Regarding the stopping of anti-epileptic drugs (AED): the 2004 NICE
guidelines recommends that can be considered if seizure free for > 2 years, with
AEDs being stopped over 2-3 months. Benzodiazepines should be withdrawn over
a longer period.
In patient who has been seizure free for more than 2 years, the chance for
recurrence in the next 2 years is 43% if they stop therapy compared with 10% if
drugs are continued. Factors that have been shown to increase the risk of re-
seizures include:
Older age.
History of tonic clonic or myoclonic seizure.
Previous abnormal brain imaging or EEG.
Use of multiple AEDs.
Seizure while on therapy.
Infantile Spasm (west syndrome):
It occurs between 3-12 months of age, managed by Vigabatrin.
It is a triad of:
1) Infantile spasm.
2) A pathognomonic EEG pattern (called hypsarrhythmia).
3) Mental retardation.
International definition requires only 2 out of these 3 elements.
Dr Khaled Magraby MRCP Notes Neurology 998
Chapter 10: Neurology
Carbamazepine: dose-dependent unwanted side effects: diplopia, nystagmus,
ataxia, hyponatremia, hepatotoxicity, Toxic Epidermal Necrolysis (TEN) and rarely
BM depression.
Epilepsy: pregnancy and breast feeding
The risks of uncontrolled epilepsy during pregnancy generally outweigh the risks of
medication to the foetus, so her drug should be continued.
There is no point in switching therapies as this could precipitate seizures in an
otherwise stable patient.
All women thinking about becoming pregnant should be advised to take folic acid 5
mg per day well before pregnancy to minimise the risk of neural tube defects.
Around 1-2% of newborns born to non-epileptic mothers have congenital defects.
This rises to 3-4% if the mother takes antiepileptic medication.
Other points:
Aim for monotherapy.
There is no indication to monitor antiepileptic drug (AED) levels.
Phenytoin: associated with cleft palate.
Sodium valproate: associated with neural tube defects.
Similarly both phenytoin and valproate are associated with teratogenic effects.
Levetiracetam: insufficient data to its use in pregnancy.
Topiramate: is associated with major congenital malformation (MCM).
Carbamazepine: often considered the least teratogenic of the older
antiepileptic.
In pregnancy total plasma concentrations of anticonvulsants fall, so the dose
of Carbamazepine may need to be increased.
The potential teratogenic effects (particularly neural tube defects) of
carbamazepine do need to be explained and in an effort to reduce this risk
she should receive folate supplements.
Screening with alpha fetoprotein (AFP) and 2nd trimester ultrasound are
required.
Lamotrigine: studies to date suggest the rate of congenital malformations
may be low. The dose of lamotrigine may need to be increased in pregnancy.
Dr Khaled Magraby MRCP Notes Neurology 999
Chapter 10: Neurology
Lowest rate of MCM were seen for Carbamazepine and Lamotrigine.
It is advised that pregnant women taking phenytoin are given vitamin K in the
last month of pregnancy to prevent clotting disorders in the newborn.
Vitamin K should be given to the mother prior to delivery.
The BNF states 'breast-feeding is acceptable and generally considered safe for
mothers with all antiepileptic drugs, taken in normal doses, with the possible
exception of barbiturates'.
Breast feeding is acceptable with nearly all anti-epileptic drugs.
Neurofibromatosis (NF)
There are two types of neurofibromatosis, NF1 and NF2.
Both are inherited in an autosomal dominant fashion.
NF1 is also known as von Recklinghausen's syndrome. It is caused by a gene
mutation on chromosome 17 which encodes neurofibromin and affects around 1 in
4,000.
NF2 is caused by gene mutation on chromosome 22 and affects around 1 in
100,000.
Features:
NF1 NF2
Café-au-lait spots (>= 6 to 15 mm in Bilateral acoustic neuromas
diameter)
Axillary/groin freckles
Iris: Lisch nodules in > 90%
Optic glioma
Scoliosis
Pheochromocytomas
Peripheral neurofibromas
A family history in only 50%
Dr Khaled Magraby MRCP Notes Neurology 1000
Chapter 10: Neurology
NF1: chromosome 17 - as neurofibromatosis has 17 characters.
NF2: chromosome 22 - all the 2's
Lisch nodules in iris are seen in more than 90%.
Patients with neurofibromatosis may develop hypertension for 3 main reasons:
1) Coexistant essential hypertension
2) Renal vascular stenosis secondary to fibromuscular dysplasia
3) Pheochromocytomas
EX: Pt. with HTN and has large cafe-au-lait spots on his trunk and some axillary
freckling (i.e. neurofibromatosis), so you should:
1) Check renal function to check diagnosis of renal vascular disease.
2) Check 24 hr. urinary collection of catecholamines to check
Pheochromocytomas.
Tuberous sclerosis (TS)
It is a genetic condition of autosomal dominant inheritance.
The responsible defects having been identified on both chromosome 9 & 16.
These chromosomes carry codes for hamartin and tuberin, protein gene products
which are responsible for regulation of cell growth.
Like neurofibromatosis, the majority of features seen in TS are neuro-cutaneous.
Cutaneous features:
1) Depigmented 'ash-leaf' spots which fluoresce under UV light.
2) Shagreen patches: roughened patches of skin over lumbar spine.
3) Adenoma sebaceum: butterfly distribution over nose.
4) Fibromata beneath nails (subungual fibromata).
5) Café-au-lait spots may be seen (of course are more commonly associated
with neurofibromatosis).
Neurological features:
1) Developmental delay
2) Intellectual impairment
3) Epilepsy (infantile spasms or partial)
Dr Khaled Magraby MRCP Notes Neurology 1001
Chapter 10: Neurology
Also:
Retinal hamartomas: dense white areas on retina (phakomata)
Rhabdomyomas of the heart
Gliomatous changes can occur in the brain lesions
Polycystic kidneys, renal angiomyolipomata
Cigarettes and coffee with a rough stupid person with a butterfly on his nose
while he is dancing.
Cigarettes (ash) – coffee (Café-au-lait spots) – stupid (Developmental and
Intellectual) – dancing (epilepsy).
Essential tremor
Essential tremor (previously called benign essential tremor) is an AD autosomal
dominant condition which usually affects both upper limbs.
Features:
Postural tremor: worse if arms outstretched (usually 6-8 Hz).
It is worse on movement and during stress.
Improved by alcohol and rest
Most common cause of titubation (head tremor)
Management:
Propranolol is first-line
Primidone (anticonvulsant) is sometimes used (as in case which cannot give
propranolol like in bronchial asthma).
NB: Anxiety and drugs (for example, salbutamol, sodium valproate,
theophylline, and amiodarone) are commonly associated with limbs tremor.
Idiopathic intracranial hypertension (IIH)
Idiopathic intracranial hypertension (also known as pseudo tumour cerebri and
formerly Benign Intracranial Hypertension (BIH)) is a condition classically seen in
young, overweight females.
Its exact cause is unknown, but it is most common in overweight young women.
Dr Khaled Magraby MRCP Notes Neurology 1002
Chapter 10: Neurology
IIH is thought to be due to impaired CSF absorption across the arachnoid villi into
the dural sinuses.
It is characterised by raised intracranial pressure with normal CSF cell count and
protein content, normal ventricular size, anatomy and position.
If left untreated it can result in permanent visual loss.
The symptoms classically reported with raised intracranial pressure are frontal
headaches, worst when lying flat, coughing, bending, and on waking as well as
nausea and vomiting. Patients may also complain of a whooshing sound in their
ears, and pain/ numbness or tingling in their arms and legs. Visual symptoms include
transient, bilateral episodes of blurred vision, transient visual field defects and double
vision.
Features:
1) Headache
2) Blurred vision
3) Fundus: Papilledema with reduced/absent retinal venous pulsation
4) Enlarged blind spot
5) Sudden loss of vision (If the optic nerve becomes compressed).
6) 6th nerve palsy may be present >> diplopia
7) Reflexes are preserved and plantars are flexor. Extensor plantars suggest an
alternative diagnosis.
8) Normal ventricles on CT and MRI
9) CSF opening pressure: mild increase (more than 20 CmH2O).
Obese, young female with headaches, blurred vision and Papilledema, ± OCP
(Dianette in PCO) but otherwise normal neurology >>> think idiopathic intracranial
hypertension (IIH)
Risk factors:
Obesity
Female sex
Pregnancy
Drugs:
1) Oral contraceptive pill,
Dr Khaled Magraby MRCP Notes Neurology 1003
Chapter 10: Neurology
2) Steroids,
3) Tetracycline (Doxycycline for acne),
4) Nitrofurantoin,
5) Vitamin A toxicity
Management:
Weight loss.
Diuretics e.g. acetazolamide
Repeated lumbar puncture or Lumbo-peritoneal (LP) shunt.
Surgery: optic nerve sheath decompression and fenestration may be
needed to prevent damage to the optic nerve.
NB: Visual loss is the single threatening complication of idiopathic intracranial
hypertension (IIH).
NB: If intracranial hypertension is thought to occur secondary to a known causes
(e.g. Medication) then it is of course not idiopathic.
EX: A 27-year-old woman is reviewed due to sudden loss of vision in her left eye.
She is known to have severe RA and is treated currently with methotrexate,
infliximab and prednisolone. She has in the past also used sulfasalazine and
hydroxychloroquine. For the past 6 weeks she has developed troublesome
headaches. Examination demonstrates bilateral papilledema. Which one of the
following is most likely to be responsible for this presentation? >> Intracranial
hypertension probably secondary to prednisolone. Patients may lose sight
suddenly if the optic nerve becomes compressed.
Migraine: diagnostic criteria
The International Headache Society has produced the following diagnostic criteria for
migraine without aura:
Dr Khaled Magraby MRCP Notes Neurology 1004
Chapter 10: Neurology
Point Criteria
A At least 5 attacks fulfilling criteria B-D
B Headache attacks lasting 4-72 hours* (untreated or unsuccessfully treated)
C Headache has at least two of the following characteristics:
1) Unilateral location*
2) Pulsating quality (i.e., varying with the heartbeat).
3) Moderate or severe pain intensity.
4) Aggravation by or causing avoidance of routine physical activity (e.g.,
walking or climbing stairs).
D During headache at least one of the following:
1) Nausea and/or vomiting*
2) Photophobia and phonophobia (occurs in around 75% of patients).
E Not attributed to another disorder (history and examination do not suggest a
secondary headache disorder or, if they do, it is ruled out by appropriate
investigations or headache attacks do not occur for the first time in close temporal
relation to the other disorder)
*In children, attacks may be shorter-lasting, headache is more commonly bilateral,
and gastrointestinal disturbance is more prominent.
N.B: Migraine with aura (around 1 in 3 migraine patients) tends to be easier to
diagnose with a typical aura being progressive in nature and may occur hours prior
to the headache. Typical aura include a transient hemianopic disturbance or a
spreading scintillating scotoma ('jagged crescent'). Sensory symptoms may also
occur.
NB: A complicated migraine is one which results in hemi sensory or hemi motor
findings associated with a typical migraine presentation i.e. it is like TIA but the
patient is young age and has past medical history for migraines.
Dr Khaled Magraby MRCP Notes Neurology 1005
Chapter 10: Neurology
Migraine: management
It should be noted that as a general rule 5-HT receptor agonists are used in the
acute treatment of migraine whilst 5-HT receptor antagonists are used in
prophylaxis.
ACute → 5HT1 AGonist
Prophylaxis → 5HT2 antagonist & βB
Migraine:
Acute: (Triptan + NSAID) or (Triptan + paracetamol).
Prophylaxis: Topiramate (Topamax®) or propranolol.
Acute treatment:
First-line: offer combination therapy with an oral triptan and an NSAID, or an
oral triptan and paracetamol.
For young people aged 12-17 years consider a nasal triptan in preference to
an oral triptan.
If the above measures are not effective or not tolerated offer a non-oral
preparation of metoclopramide or prochlorperazine and consider adding a
non-oral NSAID or triptan
Prophylaxis:
Prophylaxis should be given if patients are experiencing ≥ 2 attacks per
month. Modern treatment is effective in about 60% of patients.
NICE advice either Topiramate (Topamax®) or Propranolol 80 – 240 mg
OD according to the person's preference, comorbidities and risk of adverse
events'.
Propranolol should be used in preference to topiramate in women of child
bearing age as it may be teratogenic and it can reduce the effectiveness of
hormonal contraceptives.
If these measures fail NICE recommend 'a course of up to 10 sessions of
acupuncture over 5-8 weeks' or gabapentin.
NICE recommend: 'Advise people with migraine that riboflavin (400 mg once
a day) may be effective in reducing migraine frequency and intensity for some
people'
Dr Khaled Magraby MRCP Notes Neurology 1006
Chapter 10: Neurology
For women with predictable menstrual migraine treatment NICE recommend
either frovatriptan (2.5 mg twice a day) or zolmitriptan (2.5 mg twice or three
times a day) as a type of 'mini-prophylaxis'.
5-HT2 antagonists like:
o “Pizotifen” is no longer recommend. Adverse effects such as weight
gain & drowsiness are common.
o “Methysergide” is very rarely used as associated with retroperitoneal
fibrosis.
Avoid aspirin in children < 16 years as risk of Reye's syndrome
Codeine would also be a poor choice as it has limited benefit in migraine.
Patients with migraine experience delayed gastric emptying during acute attacks.
So decrease effect of analgesics. For this reason analgesics are often combined
prokinetic agents such as metoclopramide.
Topiramate side effects:
Renal stones
Weight loss
Cognitive impairment
Tingling in extremities.
Triptans
Triptans are specific 5-HT1 1B/1D receptors agonists used in the acute treatment of
migraine.
They are generally used first-line in combination therapy with an NSAID or
paracetamol.
Prescribing points:
Should be taken as soon as possible after the onset of headache, rather
than at onset of aura.
Oral, dispersible, nasal spray and subcutaneous injections are available.
Adverse effects:
'Triptan sensations' - tingling, heat, throat tightness and chest tightness,
heaviness, pressure.
Dr Khaled Magraby MRCP Notes Neurology 1007
Chapter 10: Neurology
Contraindications:
Patients with a history of, or significant risk factors for, IHD or cerebrovascular
disease.
Migraine: pregnancy, contraception and other hormonal factors
Migraine during pregnancy:
Paracetamol 1g is first-line
Aspirin 300mg or ibuprofen 400mg can be used second-line in the first and
second trimester.
Migraine and menstruation:
Many women find that the frequency and severity of migraines increase
around the time of menstruation.
SIGN recommends that women are treated with mefanamic acid or a
combination of aspirin, paracetamol and caffeine.
Triptans are also recommended in the acute situation, NICE recommend
either frovatriptan (2.5 mg twice a day) or zolmitriptan (2.5 mg twice or three
times a day) as a type of 'mini-prophylaxis'.
Migraine and the combined oral contraceptive (COC) pill:
If patients have migraine with aura then the COC is absolutely
contraindicated due to an increased risk of stroke (relative risk 8.72).
Ergot derived compounds and triptans are contraindicated for the ttt of
hemiplegic migraine because of the risk of precipitating a stroke.
Migraine and hormone replacement therapy (HRT):
Safe to prescribe HRT for patients with a history of migraine but it may make
migraines worse.
Cluster headache (CH)
Cluster headaches are more common in men (10:1) and smokers.
Features:
Pain typical occurs once or twice a day, each episode lasting 15 mins - 2
hours.
Clusters typically last 4-12 weeks.
Intense pain around one eye (recurrent attacks 'always' affect same side).
Dr Khaled Magraby MRCP Notes Neurology 1008
Chapter 10: Neurology
Accompanied by eye redness, lacrimation, lid swelling
Miosis and ptosis in a minority.
Nasal stuffiness, Rhinorrhoea.
Patient is restless during an attack
Examination between the attacks should be normal.
Episodic eye pain, lacrimation, nasal stuffiness occurring daily >>> cluster headache
Management:
Acute: 100% oxygen, SC Sumatriptan, nasal lidocaine.
Prophylaxis: Verapamil, prednisolone.
Consider specialist referral
NB: Sumatriptan has been associated with chest pain in up to 17% of cases,
possibly due to vasospasm. Also it is associated with MI and so contraindicated in
people with known IHD.
D.D: Chronic paroxysmal hemicrania (CPH) has features of cluster headaches but
is associated with attacks of shorter duration and increased frequency e.g. each
headache can last between 3-45 minutes and occur 20-40 times per day. It
responds very well to indomethacin.
N.B: Some neurologists use the term trigeminal autonomic cephalgia to group a
number of conditions including cluster headache (CH), chronic paroxysmal
hemicrania (CPH) and short-lived unilateral neuralgiform headache with conjunctival
injection and tearing (SUNCT).
Pituitary apoplexy
Sudden enlargement of pituitary tumour secondary to haemorrhage or infarction. A
pituitary adenoma usually pre-exists.
Endocrinologically, the main initial problem is a lack of adrenocorticotropic
hormone (ACTH), which results in a lack of cortisol and the features of an
'Addisonian crisis', i.e. hypotension, hyponatraemia, hyperkalaemia and
hypoglycaemia. Subacutely, there can be deficiency in thyroid stimulating hormone
(TSH) and gonadotropins (LH and FSH).
Dr Khaled Magraby MRCP Notes Neurology 1009
Chapter 10: Neurology
Features:
1) Sudden onset headache similar to that seen in SAH.
2) Vomiting
3) Neck stiffness
4) Visual field defects: classically bitemporal superior quadrantic defect.
5) Extra ocular nerve palsies in up to 80%, with III nerve palsy the commonest
finding.
6) Features of pituitary insufficiency e.g. Hypotension secondary to
hypoadrenalism.
MRT pituitary to confirm the diagnosis
TTT: Hydrocortisone IV should be given to prevent addisonian crisis.
EX: A 47-year-old man presents to the Emergency Department with a 3 day history
of severe headache associated with vomiting. There is no past medical history of
note. On examination blood pressure is 98/62 mmHg, pulse is 108 bpm and
temperature is 37.0ºC. There is mild neck stiffness and a partial third nerve palsy of
the left eye. Blood rests reveal: Na+ = 130, K= 5.2, Low free T4. The most likely
diagnosis is >> Pituitary apoplexy.
Medication overuse headache
It is one of the most common causes of chronic daily headache.
It may affect up to 1 in 50 people.
Features:
Present for ≥ 15 days per month.
Developed or worsened whilst taking regular symptomatic medication.
Patients using opioids and Triptans are at most risk.
May be psychiatric co-morbidity.
Management (SIGN guidelines):
Simple analgesics and Triptans should be withdrawn abruptly (may initially
worsen headaches).
Opioid analgesics should be gradually withdrawn.
Dr Khaled Magraby MRCP Notes Neurology 1010
Chapter 10: Neurology
Medication overuse headache:
Simple analgesia + Triptans: stop abruptly
Opioid analgesia: withdraw gradually
Acoustic neuroma
Acoustic neuromas (more correctly called vestibular schwannomas: it is a benign
primary intracranial tumour of the myelin forming cells of the vestibulocochlear
nerve CN VIII).
It accounts for approximately 5% of intracranial tumours and 90 % of
cerebellopontine angle.
The term “acoustic” is a misnomer as the tumour rarely arises from the acoustic (or
cochlear) division of the vestibulocochlear nerve.
Features can be predicted by the affected cranial nerves:
Cranial nerve (5) V: absent corneal reflex
Cranial nerve (7) VII: facial palsy
Cranial nerve (8) VIII: hearing loss, vertigo, tinnitus
Bilateral acoustic neuromas are seen in neurofibromatosis type 2.
MRI of the cerebellopontine angle is the investigation of choice.
It is a benign neoplasm and the lesion can be resected with a good prognosis
Loss of corneal reflex → think of Acoustic neuroma
Benign paroxysmal positional vertigo (BPPV)
It is one of the most common causes of vertigo encountered.
It is characterised by the sudden onset of dizziness and vertigo triggered by
changes in head position.
The average age of onset is 55 years and it is less common in younger patients.
Features:
Vertigo triggered by change in head position (e.g. rolling over in bed or gazing
upwards).
Dr Khaled Magraby MRCP Notes Neurology 1011
Chapter 10: Neurology
May be associated with nausea.
Each episode typically lasts 10-20 seconds.
Positive Dix-Hallpike manoeuvre.
BPPV has a good prognosis and usually resolves spontaneously after a few weeks
to months. Symptomatic relief may be gained by:
Epley manoeuvre (successful in around 80% of cases)
Teaching the patient exercises they can do themselves at home, for example
Brandt-Daroff exercises.
Medication is often prescribed e.g. Betahistine (Betaserc ®) but it tends to be
of limited value.
Meniere's disease
It is a disorder of the inner ear of unknown cause.
It is characterised by excessive pressure and progressive dilation of the
endolymphatic system.
It is more common in middle-aged adults but may be seen at any age.
Meniere's disease has a similar prevalence in both men and women.
Features:
Recurrent episodes of vertigo, tinnitus and hearing loss (sensorineural, i.e.
+ve Weber's test).
Vertigo is usually the prominent symptom.
A sensation of aural fullness or pressure is now recognised as being
common.
Other features include nystagmus and a positive Romberg test
Episodes last minutes to hours.
Typically symptoms are unilateral but bilateral symptoms may develop after a
number of years.
Natural history:
Symptoms resolve in the majority of patients after 5-10 years
Some patients may be left with hearing loss.
Psychological distress is common.
Dr Khaled Magraby MRCP Notes Neurology 1012
Chapter 10: Neurology
Management:
ENT assessment is required to confirm the diagnosis.
Patients should inform the DVLA. The current advice is to cease driving until
satisfactory control of symptoms is achieved.
Acute attacks:
o Buccal or intramuscular Prochlorperazine.
o Admission is sometimes required.
Prevention: Betahistine may be of benefit.
Tinnitus (causes):
Meniere's Associated with hearing loss, vertigo, tinnitus and sensation of
disease fullness or pressure in one or both ears.
Otosclerosis Onset is usually at 20-40 years.
Conductive deafness.
Tinnitus.
Normal tympanic membrane. (10% of patients may have a
'flamingo tinge', caused by hyperaemia).
Positive family history.
Acoustic Hearing loss, vertigo, tinnitus
neuroma Absent corneal reflex is important sign
Associated with neurofibromatosis type 2
Hearing loss Causes include excessive loud noise and presbycusis
Drugs Aspirin
Aminoglycosides
Lasix
Quinine
Other causes Impacted ear wax
Chronic suppurative otitis media (CSOM)
Dr Khaled Magraby MRCP Notes Neurology 1013
Chapter 10: Neurology
Rinne's and Weber's test
Performing both to allows differentiation of conductive and sensorineural deafness.
Rinne's test:
Tuning fork is placed over the mastoid process until the sound is no longer
heard, followed by repositioning just over external acoustic meatus.
Air conduction (AC) is normally better than bone conduction (BC).
If BC > AC then conductive deafness.
Weber's test:
Tuning fork is placed in the middle of the forehead equidistant from the
patient's ears.
The patient is then asked which side is loudest.
In unilateral sensorineural deafness, sound is localised to the unaffected
side.
In unilateral conductive deafness, sound is localised to the affected side.
EX: Weber's test: If sound is localized to the Rt. side: so either Rt. conductive
deafness or Lt sensorineural deafness.
Intracranial venous thrombosis
Overview:
Can cause cerebral infarction, much less common than arterial causes.
50% of patients have isolated sagittal sinus thrombosis - the remainder
have coexistent lateral sinus thrombosis and cavernous sinus thrombosis.
Venous sinus thrombosis is associated with the oral contraceptive pill, the
post-partum period and other hypercoagulable states.
Features:
Headache (may be sudden onset)
Nausea & vomiting
Sagittal sinus thrombosis:
May present with seizures and hemiplegia
Parasagittal biparietal or bifrontal haemorrhagic infarctions are sometimes
seen
Dr Khaled Magraby MRCP Notes Neurology 1014
Chapter 10: Neurology
Cavernous sinus thrombosis:
1) Other causes of cavernous sinus syndrome: local infection (e.g. sinusitis),
neoplasia, trauma
2) Periorbital oedema.
3) Ophthalmoplegia: 6th nerve damage typically occurs before 3rd & 4th.
4) Trigeminal nerve (5th) involvement may lead to hyperaesthesia of upper
face and eye pain.
5) Central retinal vein thrombosis.
Lateral sinus thrombosis:
6th and 7th cranial nerve palsies.
EX: A 28-year-old female, 3 days post-partum, develops severe headache
associated with seizures.
During her pregnancy her BP had been mildly elevated in the third trimester.
On examination, she had a GCS of 15 but was slightly confused and drowsy. Her
temperature was 37.5°C, she had mild nuchal rigidity but neurological examination
was otherwise normal.
The most likely diagnosis >>> Cortical vein thrombosis >>> Do MRV.
TTT is Anti-coagulation
NB:
- Post-partum period is a risk factor of cortical vein and sinus thrombosis.
- It typically presents with headache, seizures and focal neurological deficit 2 to 3
weeks postpartum (but is also seen earlier).
- Although eclampsia would be in the differential diagnosis in this case, nuchal
rigidity is not a typical feature which points more to the direction of cortical vein
thrombosis.
- The eclampsia seizures can in fact occur antepartum, intrapartum, or postpartum.
If a seizure does occur postpartum, it usually occurs within the first 24 hours after
delivery.
- Other clinical presentations include an idiopathic intracranial hypertension (BIH)
type of picture (papilledema, visual disturbances and headaches) or a subacute
encephalopathic picture. Thrombophilia screen should be performed.
Dr Khaled Magraby MRCP Notes Neurology 1015
Chapter 10: Neurology
EX: A 30-year-old lady presented with headache which she first noticed as she was
picking up her 5-week-old baby. صداع وزغللة من النور لسيدة بعد الوالدة
On admission she was unable to tolerate the lights and complained of feeling sick.
Fundoscopy showed bilateral papilledema, and she was complaining that she was
unable to see on her left side. CT head showed a small right occipital bleed >>>
venous sinus thrombosis >>> TTT: urgent LMWH.
This lady has developed a venous sinus thrombosis peri-partum resulting in
symptoms of headache, photophobia and vomiting.
The small occipital bleed has resulted from venous congestion and she needs urgent
ttt with LWMH to prevent clot propagation and further complications.
This woman's symptoms are not as a result of a subarachnoid aneurysm, so
aneurysm coiling and nimodipine are not appropriate.
A lumbar puncture is not appropriate as she does not have idiopathic intracranial
hypertension.
She does not have migraine and so a triptan is not appropriate.
Altitude related disorders
There are three main types of altitude related disorders:
1) Acute mountain sickness (AMS), which may progress to
2) High altitude pulmonary oedema (HAPE) or
3) High altitude cerebral oedema (HACE).
All are due to the chronic hypobaric hypoxia which develops at high altitudes.
AMS is generally a self-limiting condition.
Features of AMS start to occur above 2,500 - 3,000m, developing gradually over 6-
12 hours and potentially last a number of days:
Headache
Nausea
Fatigue
Prevention and treatment of AMS:
The risk of AMS may actually be positively correlated to physical fitness.
Gain altitude at no more than 500 m per day.
Dr Khaled Magraby MRCP Notes Neurology 1016
Chapter 10: Neurology
Acetazolamide (a carbonic anhydrase inhibitor) is widely used to prevent
AMS and has a supporting evidence base.
Treatment: descent.
A minority of people above 4,000m go onto develop high altitude pulmonary oedema
(HAPE) or high altitude cerebral oedema (HACE), potentially fatal conditions:
HAPE presents with classical pulmonary oedema features
HACE presents with headache, ataxia, Papilloedema
Management of HAPE:
Descent
Oxygen if available
Nifedipine, Dexamethasone, Acetazolamide, phosphodiesterase type V
inhibitors*
Management of HACE:
Descent
Dexamethasone
*The relative merits of these different treatments has only been studied in small
trials. All seem to work by reducing systolic pulmonary artery pressure
DVLA: neurological disorders
The guidelines below relate to car/motorcycle use unless specifically stated. For
obvious reasons, the rules relating to drivers of heavy goods vehicles tend to be
much stricter.
Specific rules:
First seizure: 6 months off driving*.
For patients with established epilepsy they must be fit free for 12 months
before being able to drive.
Stroke or TIA: 1 month off driving
Multiple TIAs over short period of times: 3 months off driving
Craniotomy e.g. For meningioma: 1 year off driving**
Pituitary tumour: craniotomy: 6 months; trans-sphenoidal surgery 'can drive
when there is no debarring residual impairment likely to affect safe driving'
Dr Khaled Magraby MRCP Notes Neurology 1017
Chapter 10: Neurology
Narcolepsy/cataplexy: cease driving on diagnosis, can restart once
'satisfactory control of symptoms'.
Chronic neurological disorders e.g. multiple sclerosis, motor neuron disease:
DVLA should be informed, complete PK1 form (application for driving licence
holder’s state of health).
Syncope:
Simple faint: no restriction
Single episode, explained and treated: 1 month off
Single episode, unexplained: 6 months off
Two or more episodes: 12 months off
*Previously rule was 12 months. It is now 6 months off driving if the licence holder
has undergone assessment by an appropriate specialist and no relevant abnormality
has been identified on investigation, for example EEG and brain scan where
indicated.
**If the tumour is a benign meningioma and there is no seizure history, licence can
be reconsidered 6 months after surgery if remains seizure free.
Motor neuron disease (MND): types
MND is a neurological condition of unknown cause which can present with both
UMNL and LMNL signs.
There are a number of clues which point towards a diagnosis of MND:
1) LMN signs in arms and UMN signs in legs
2) Fasciculation
3) Wasting of the small hand muscles/tibialis anterior is common.
4) 10% of patients with MND have dementia (front temporal) (FTD).
5) NO sensory affection (vague sensory symptoms may occur early in the
disease (e.g. limb pain) but 'never' sensory signs).
6) NOT external ocular muscles affection.
7) NO cerebellar signs.
8) Abdominal reflexes are usually preserved and sphincter dysfunction if present
is a late feature.
‘Fasciculations’ >>> think MND, most commonly Amyotrophic lateral sclerosis.
Dr Khaled Magraby MRCP Notes Neurology 1018
Chapter 10: Neurology
It rarely presents before 40 years and various 4 patterns of disease are
recognised including: amyotrophic lateral sclerosis, primary lateral sclerosis,
progressive muscular atrophy and bulbar palsy.
In some patients however, there is a combination of clinical patterns.
1) Amyotrophic lateral sclerosis (ALS):
The most common type of MND (50% of patients).
Typically LMN signs in arms and UMN signs in legs.
In familial cases the gene responsible lies on chromosome 21 and codes for
superoxide dismutase.
2) Primary lateral sclerosis:
UMN signs only.
3) Progressive muscular atrophy:
LMN signs only.
Affects distal muscles before proximal
Carries best prognosis
4) Progressive bulbar palsy:
Palsy of the tongue, muscles of chewing/swallowing and facial muscles due to
loss of function of brainstem motor nuclei.
Carries worst prognosis.
Diagnosis:
The diagnosis of motor neuron disease is clinical, but nerve conduction studies will
show normal motor conduction and can help exclude a neuropathy.
EMG (Electromyography) shows a ↓ number of action potentials with a ↑ amplitude.
MRI is usually performed to exclude the differential diagnosis of cervical cord
compression and myelopathy.
Dr Khaled Magraby MRCP Notes Neurology 1019
Chapter 10: Neurology
Management:
1) Riluzole (Rilutek®):
Prevents stimulation of glutamate receptors (Anti-glutamate).
Used mainly in amyotrophic lateral sclerosis.
Prolongs life by about 3 months.
Expensive.
2) Respiratory care:
NIV (Non-invasive ventilation, usually BIPAP) is used at night.
Studies have shown that NIV has a survival benefit of around 7 months.
MND (Motor neuron disease) >>> ttt: NIV is better than Riluzole.
Prognosis: poor: 50% of patients die within 3 years.
Myotonic dystrophy
Myotonic dystrophy (also called dystrophia myotonica, DM) is an inherited
myopathy with features developing at around 20-30 years old.
It is the most common adult muscular dystrophy.
It affects skeletal, cardiac and smooth muscle + eyes, endocrine, CNS.
There are 2 main types of myotonic dystrophy, DM1 and DM2.
Genetics:
Autosomal dominant
A trinucleotide repeat disorder
DM1 is caused by a CTG repeat at the end of the DMPK (Dystrophia
Myotonica-Protein Kinase) gene on chromosome 19.
DM2 is caused by a repeat expansion of ZNF9 gene on chromosome 3.
The key differences are listed in table below:
Dr Khaled Magraby MRCP Notes Neurology 1020
Chapter 10: Neurology
DM1 DM2
- DMPK gene on chromosome 19. - ZNF9 gene on chromosome 3.
- Distal weakness more prominent. - Proximal weakness more prominent.
- Severe congenital form not seen.
General features:
1) Myotonia (tonic spasm of muscle) (slow-relaxing grip may be noticed on
initial hand-shake with the patient and is typical of myotonic dystrophy).
2) Myotonic facies (long, 'haggard' appearance).
3) Atrophy of temporalis, masseters, facial muscle.
4) Frontal balding
5) Bilateral ptosis
6) Neck muscles, including sternocleidomastoid, are involved early in the
course of disease.
7) Cataracts >> lead to loss of red reflexes of eyes.
8) Dysarthria (secondary to myotonia of the tongue and pharynx).
9) Dysphagia
Other features:
1) Reduced reflexes with myotonia
2) Weakness of arms and legs (distal initially)
3) Mild mental / Intellectual impairment
4) DM
5) Testicular atrophy
6) Cardiac involvement: heart block (1 st degree HB a prolonged PR interval is
seen in around 20-40% of patients) or (CHB), and cardiomyopathy.
Dr Khaled Magraby MRCP Notes Neurology 1021
Chapter 10: Neurology
Dystrophia myotonica - DM1
Autosomal Dominant
Distal weakness initially
Diabetes
Dysarthria
Dysphagia
Diagnosis can be made on electromyogram (EMG) and muscle biopsy.
EX: A 29-year-old female with weakness in her arms, leading to increasing
difficulties at work. On examination she has a bilateral ptosis and loss of the red-
reflex in both eyes. Urine testing also reveals glycosuria. What is the most likely
diagnosis? >> Dystrophia myotonica - DM1.
EX: A 27-year-old female presents complaining of generalised weakness.
Examination of her face reveals bilateral ptosis, dysarthric speech and a slow-
relaxing grip. What is the most likely diagnosis? >>Dystrophia myotonica - DM1.
TTT:
Ankle-foot orthosis and splints: for foot drop.
Lid-lifting surgery has no place as the ptosis is progressive, except in
severe cases; also ± cataract extraction.
Dr Khaled Magraby MRCP Notes Neurology 1022
Chapter 10: Neurology
Dermatomes
Nerve
root Landmark Mnemonics
C2 Posterior half of the skull (cap)
C3 High turtleneck shirt
C4 Low-collar shirt
C5, C6 Thumb + index finger Make a 6 with your L hand by touching the tip of
the thumb & index finger together - C6
C7 Middle finger + palm of hand
C8 Ring + little finger
T4 Nipples T4 at the Teat Pore
T5 Inframammary fold
T7 Xiphoid process
T10 Umbilicus BellybuT-TEN
T12 Pubic bone region
L1 Inguinal ligament L for ligament, 1 for 1nguinal
L4 Knee caps Down on aLL fours - L4
L5 Big toe, dorsum of foot (except L5 = Largest of the 5 toes
lateral aspect)
S1 Lateral foot, small toe S1 = the smallest one
S2, S3 Genitalia
Dr Khaled Magraby MRCP Notes Neurology 1023
Chapter 10: Neurology
Dr Khaled Magraby MRCP Notes Neurology 1024
Chapter 10: Neurology
Restless legs syndrome (RLS)
It is a syndrome of spontaneous, continuous lower limb movements that may be
associated with paraesthesia.
It is extremely common, affecting between 2-10% of the general population.
Males and females are equally affected and a family history may be present.
Clinical features:
Uncontrollable urge to move legs (akathisia). Symptoms initially occur at
night but as condition progresses may occur during the day. Symptoms are
worse at rest.
Paraesthesias e.g. 'crawling' or 'throbbing' sensations.
Movements during sleep may be noted by the partner - periodic limb
movements of sleeps (PLMS).
Causes and associations:
There is a positive family history in 50% of patients with idiopathic RLS.
Iron deficiency anaemia.
Uraemia
DM
Pregnancy
The diagnosis is clinical although bloods to exclude iron deficiency anaemia may
be appropriate (as Ferritin).
Management:
Simple measures: walking, stretching, massaging affected limbs.
Treat any iron deficiency.
Dopamine agonists are first-line treatment (e.g. Ropinirole, Pramipexole,
and Rotigotine).
Benzodiazepines.
Gabapentin.
EX: A 67-year-old woman comes for review with her husband. Her husband
complains that she is constantly getting up from bed at night and pacing around the
bedroom. She complains of 'antsy' legs and a 'horrible, creeping sensation'. Her
Dr Khaled Magraby MRCP Notes Neurology 1025
Chapter 10: Neurology
symptoms generally come on in the evening and are only relieved by moving round.
>>> Restless legs syndrome (RLS) >> ttt: Ropinirole.
Chorea
Chorea describes involuntary, rapid, jerky movements which often move from one
part of the body to another.
Chorea is caused by damage to the basal ganglia, especially the caudate nucleus.
Slower, sinuous movement of the limbs is termed athetosis.
Causes of chorea:
1) Huntington's disease, Wilson's disease, Ataxic telangiectasia.
2) Rheumatic fever: Sydenham's chorea.
3) SLE, anti-phospholipid syndrome, vasculitis (PAN, Behcet's disease).
4) Drugs: oral contraceptive pill, L-dopa, antipsychotics.
5) Pregnancy: chorea gravidarum.
6) Thyrotoxicosis.
7) Neuro-acanthocytosis.
8) Polycythaemia rubra vera.
9) CO poisoning, cyanide, opiates, mercury.
10) Cerebrovascular disease.
Tetrabenazine works as a VMAT-inhibitor (vesicular monoamine transporter-2),
involved in transportation of monoamines. It is indicated for Huntington's chorea to
reduce hyperkinetic movements.
Ataxic telangiectasia
Basics:
Autosomal recessive disorder
Combined immunodeficiency disorder
Features:
1) Cerebellar ataxia
2) Telangiectasia
Dr Khaled Magraby MRCP Notes Neurology 1026
Chapter 10: Neurology
3) Recurrent chest infections
4) 10% risk of developing malignancy, lymphoma or leukaemia, but also non-
lymphoid tumours
Hemiballism
Hemiballism occurs following damage to the subthalamic nucleus.
Ballisic movements are involuntary, sudden, jerking, severe flinging movements
following no particular pattern which occur contralateral to the side of the lesion.
The ballisic movements primarily affect the proximal limb musculature whilst the
distal muscles may display more choreiform-like movements.
Symptoms may decrease whilst the patient is asleep.
Aetiology:
* Stroke in elderly, with arteriosclerotic risk factors including HTN, DM, IHD,
dyslipidaemia.
* Infection or inflammatory in young.
Anti-dopaminergic agents (e.g. Haloperidol) are the mainstay of treatment.
Von Hippel-Lindau syndrome (VHL)
VHL syndrome is an autosomal dominant condition predisposing to neoplasia.
It is due to an abnormality in the VHL gene on short arm of chromosome 3.
Features
1) Cerebellar haemangiomas (Ataxia).
2) Retinal haemangiomas: are bilateral in 25% of patients and may lead to
vitreous haemorrhage.
3) Renal cysts (premalignant).
4) Extra-renal cysts: epididymis, pancreatic, hepatic.
5) Phaeochromocytoma.
6) Endolymphatic sac tumours.
Dr Khaled Magraby MRCP Notes Neurology 1027
Chapter 10: Neurology
Friedreich's ataxia
Friedreich's ataxia is the most common of the early-onset hereditary ataxias.
It is an autosomal recessive, trinucleotide repeat disorder characterised by a GAA
repeat in the X25 gene on chromosome 9 (frataxin).
Neurodegeneration of cerebellum, Pyramidal tracts and dorsal root ganglia.
So there will be cerebellum dysfunction, spastic paraparesis and absent
reflexes in lower limbs.
Friedreich's ataxia is unusual amongst trinucleotide repeat disorders in not
demonstrating the phenomenon of anticipation.
The typical age of onset is 10-15 years old.
Neurological features:
1) Gait ataxia and Kyphoscoliosis ± bilateral pes cavus are the most common
presenting features.
2) Cerebellar ataxia (Dysarthria, Nystagmus).
3) Optic atrophy
4) Pyramidal weakness:
5) Extensor plantars response with absent ankle jerks.
6) Sensory-motor neuropathy
7) Spinocerebellar tract degeneration
Other features:
1) HOCM Hypertrophic obstructive cardiomyopathy (90%, most common cause
of death)
2) DM Diabetes mellitus (10-20%)
3) High-arched palate
Dr Khaled Magraby MRCP Notes Neurology 1028
Chapter 10: Neurology
Wernicke's encephalopathy
Wernicke's encephalopathy is a neuropsychiatric disorder caused by thiamine
deficiency which is most commonly seen in alcoholics.
It is caused by lesions in the medial thalamic nuclei, mammillary bodies,
periaqueductal and periventricular brainstem nuclei and superior cerebellar vermis.
Rarer causes include: persistent vomiting, stomach cancer, dietary deficiency and
after bariatric surgery.
A classic triad of Confusion, ataxia (nystagmus), and Ophthalmoplegia.
In Wernicke's encephalopathy petechial haemorrhages occur in a variety of
structures in the brain including the mammillary bodies and ventricle walls.
Features (wide variety of presentations):
1) Confusion, altered GCS.
2) Nystagmus (the most common ocular sign).
3) Ataxia.
4) Ophthalmoplegia.
5) Peripheral sensory neuropathy.
6) Impairment of short term memory.
Investigations:
1) Decreased red cell transketolase.
2) MRI.
TTT: It is a medical emergency, requiring urgent IV thiamine 50 mg IV.
The episode of Wernicke's encephalopathy may be precipitated by IV dextrose
administration which has exhausted his vitamin B reserves, hence B vitamins must
be administered to all alcoholic patients requiring dextrose.
[
Alcohol is a common cause of hypoglycaemia, and can be rapidly life-threatening if
not recognised. Common initial symptoms are tachycardia and sweating. RBS
should be checked in all patients who become acutely unwell.
Dr Khaled Magraby MRCP Notes Neurology 1029
Chapter 10: Neurology
N.B: Inability to acquire new memories with subsequent compensatory
confabulation تسامرby the patient >>> suggests the development of Korsakoff’s
amnesic syndrome:
Other symptoms may include: Delirium, Anxiety, Fear, Depression, Confusion,
Delusions, Insomnia, Painful extremities, sometimes bilateral wrist drop but more
frequently bilateral foot drop with pain or pressure over the long nerves.
Marchiafava Bignami Syndrome: Corpus callosum degeneration from chronic
alcohol excess.
Multiple Sclerosis (MS)
It is an UMNL.
It is also known as disseminated sclerosis or encephalomyelitis disseminate.
It is an autoimmune disease in which the body’s immune system attacks his own
CNS (brain and spinal cord) leading to demyelination of the CNS and hence
UMNL signs are seen.
Onset usually occurs in young adults, more common in females.
Patients with multiple sclerosis may present with non-specific features, for example
around 75% of patients have significant lethargy.
Remember that MS affects the CNS, whereas SCD subacute combined
degeneration of the cord affects central and peripheral nerves.
Remember that MS is a central nervous system (CNS) disorder causing UMNL
findings only.
NB: Fasciculations are a LMNL finding hence MS cannot be the cause of the
patient's fasciculations.
Subtypes:
Relapsing-remitting: characterized by unpredictable relapse followed by periods of
months to years of relative quiet (remission) with no new signs of disease activity.
Deficits suffered during attacks may either resolve or leave sequelae. This describes
the initial course of 85-90% of individuals with MS. When deficits always resolve
between attacks, it is referred to as benign MS.
Dr Khaled Magraby MRCP Notes Neurology 1030
Chapter 10: Neurology
Primary progressive: about 10-15% of individuals who never have remission after
their initial MS symptoms. It is characterized by progression of disability from onset
with no or only occasional minor remission and improvements. The age of onset of
that type is later than other types.
Secondary progressive: those with initial relapsing remission MS who then begin to
have progressive neurological decline between acute attacks without any definite
periods of remission. The median time between disease onset and conversion from
relapsing-remission to secondary progressive type is 19 years.
Progressive relapsing: those who from onset have a steady neurologic decline but
also suffer clear superimposed attacks. It the least common type.
Non-standard behaviours
Features:
Visual:
1) Optic neuritis: common presenting feature
2) Optic atrophy
3) Central scotoma
4) Uhthoff's phenomenon: worsening of vision following rise in body
temperature (e.g. during a hot bath). ضعف النظر بعد دش حمام دافىء
5) Internuclear ophthalmoplegia.
Motor:
Spastic weakness: most commonly in the legs (NO HYPOTONIA).
Patellar clonus
Positive plantars
Brisk reflexes
Sensory:
1) Pins/needles
2) Numbness
3) Trigeminal neuralgia
4) Lhermitte's syndrome: Paraesthesias in limbs on neck flexion
Dr Khaled Magraby MRCP Notes Neurology 1031
Chapter 10: Neurology
Cerebellar:
Ataxia: more often seen during an acute relapse than as a presenting
symptom
Tremor
Others:
Urinary incontinence/ retention
Sexual dysfunction
Intellectual deterioration
Diagnosis:
The gold standard for diagnosis of MS remains clinical assessment, with evidence of
white matter symptoms and signs disseminated in time and space.
MRI is the investigation which will show periventricular white matter plaques of
different ages and in different locations, to add weight to the clinical assessment.
CT brain will not show these changes.
Diagnosis: (demonstration of lesions disseminated in time and place):
1) MRI (Brain & Spinal cord):
i. High signal T2 lesions.
ii. Periventricular plaque.
2) CSF:
i. Oligoclonal bands (not in serum)
ii. ↑ Intrathecal synthesis of IgG
3) VEP:
i. Delayed, but well preserved wave form.
Treatment in multiple sclerosis:
It is focused at reducing the frequency and duration of relapses. There is no cure.
Acute relapse:
High dose steroids (e.g. IV Methylprednisolone) may be given for 3-5 days to
shorten the length of an acute relapse.
It should be noted that steroids shorten the duration of a relapse and do not alter the
degree of recovery (i.e. whether a patient returns to baseline function).
Dr Khaled Magraby MRCP Notes Neurology 1032
Chapter 10: Neurology
Disease modifying drugs:
Beta-interferon has been shown to reduce the relapse rate by up to 30%. Certain
criteria have to be met before it is used:
Relapsing-remitting disease + 2 relapses in past 2 years + able to walk 100m
unaided.
Secondary progressive disease + 2 relapses in past 2 years + able to walk
10m (aided or unaided).
Reduces number of relapses and MRI changes, however doesn't reduce
overall disability.
Beta-interferon can reduce the relapse rate by approximately one third for the first 2
years of treatment.
There are 3 products used, beta-interferon 1a (Avonex and Rebif), both of which are
licensed for relapsing-remitting MS and beta-interferon 1b (Betaferon), licensed for
both relapsing remitting and for secondary progressive forms of MS.
Patient should fulfil the following criteria from the Association of British Neurologists
(ABN) for commencing beta-interferon therapy:
1) Had more than 2 separate episodes within the last 2 years.
2) Is more than 18-years-old, and
3) Can walk more than 100 metres.
Contraindications to beta-interferon are:
1) History of severe clinical depression
2) Uncontrolled epilepsy
3) Hepatic dysfunction, and
4) Myelosuppression.
Other drugs used in the management of multiple sclerosis include:
Glatiramer acetate: immunomodulating drug - acts as an 'immune decoy'.
Natalizumab: a recombinant monoclonal antibody that antagonises Alpha-4
Beta-1-integrin found on the surface of leucocytes, thus inhibiting migration of
leucocytes across the endothelium across the blood-brain barrier into
parenchymal tissue.
Fingolimod: sphingosine 1-phosphate receptor modulator, it has also been
reported to be a cannabinoid receptor antagonist as well as a ceramide
synthase inhibitor. It is an immunomodulator, which prevents lymphocytes
from leaving lymph nodes. An oral formulation is available.
Dr Khaled Magraby MRCP Notes Neurology 1033
Chapter 10: Neurology
Some specific problems:
Spasticity:
Baclofen and gabapentin are first-line.
Other options include diazepam, Dantrolene and tizanidine.
Physiotherapy is important
Cannabis and Botox are undergoing evaluation.
Hallucinations are occasionally seen on the baclofen withdrawal.
Bladder dysfunction:
May take the form of urgency, incontinence, overflow etc.
Guidelines stress the importance of getting an ultrasound first to assess
bladder emptying - anticholinergics may worsen symptoms in some patients.
If significant residual volume → intermittent self-catheterisation.
If no significant residual volume → anticholinergics may improve urinary
frequency.
Good prognosis features:
1) Female sex
2) Young age of onset
3) Relapsing-remitting disease
4) Sensory symptoms
5) Long interval between first two relapses
Ways of remembering prognostic features that: the typical patient carries a better
prognosis than an atypical presentation.
Guillain-Barre syndrome (GBS): (Post-infectious polyradiculopathy)
It is an immune-mediated acute inflammatory demyelinating polyneuropathy.
GBS describes an immune mediated demyelination of the peripheral nervous
system often triggered by an infection such as viral (CMV), viral pneumonia or
bacterial (classically Campylobacter jejuni).
GBS is a post-infectious acute polyneuritis typified by elevated CSF protein with
few cells and often normal glucose.
Dr Khaled Magraby MRCP Notes Neurology 1034
Chapter 10: Neurology
Pathogenesis:
Cross reaction of antibodies with gangliosides in the peripheral nervous
system.
Correlation between anti-ganglioside antibody (e.g. anti-GM1) and clinical
features has been demonstrated.
Anti-GM1 antibodies in 25% of patients.
The characteristic features of GBS is progressive weakness of all four limbs.
The weakness is classically ascending i.e. the lower extremities are affected first,
however it tends to affect proximal muscles earlier than the distal ones.
Ascending symmetrical progressive weakness and numbness affecting initially
the legs and extending to involve the upper limb.
Weakness begins in the legs and progressively ascends to involve the trunk, upper
limbs and finally the bulbar muscles (Landry's ascending paralysis).
Asymmetry is present in only 9% of patients, with symmetrical involvement being
typical.
Sensory symptoms tend to be mild (e.g. distal paraesthesia, peripheral sensory
neuropathy) with very few sensory signs (NO sensory loss).
Usually there is painless progression over days or weeks, but in cases of abrupt
onset, there may be tenderness or muscle pain.
Some patients experience back pain in the initial stages of the illness.
Other features:
Areflexia (All tendon reflexes were absent and the plantar responses were
flexor).
Cranial nerve involvement e.g. diplopia
Autonomic dysfunction: e.g. urinary retention and postural hypotension&
tachycardia.
CSF protein is elevated to more than twice the upper limit of normal, with
normal glucose and no pleocytosis. Bacterial cultures are negative and viral
cultures rarely isolate anything.
The dissociation between a high CSF protein, normal glucose and a lack
of CSF cellular response in a person with an acute or subacute
polyneuropathy is diagnostic of Guillain-Barre syndrome.
Dr Khaled Magraby MRCP Notes Neurology 1035
Chapter 10: Neurology
Although LP is an important part of GBS assessment, a normal CSF protein
does not exclude the condition.
Less common findings:
Papilloedema: thought to be secondary to reduced CSF resorption.
EX: A 50-year-old woman with a 2 week history of difficulty walking and weakness in
her arms.
O/E, there was proximal and distal limb weakness which was more marked in the
legs than the arms. All tendon reflexes were absent and the plantar responses
were flexor. There was no sensory loss.
BP in the supine position was 140/78 mmHg & was 110/70 mmHg on standing.
What is the most likely diagnosis >>> classical presentation of Guillain-Barre
Complications include:
Bulbar involvement occurs in 50%, with a risk of aspiration and respiratory
insufficiency which can be problematic.
In 20% of cases there is urinary incontinence or retention.
Miller Fisher syndrome:
It is a variant of Guillain-Barre syndrome (GBS)
Associated with ophthalmoplegia and ataxia.
The eye muscles are typically affected first.
Usually presents as a descending paralysis rather than ascending as seen in
other forms of Guillain-Barre syndrome.
Anti-GQ1b antibodies are present in 95% of cases (highly specific).
Bickerstaff's encephalitis is like Miller-Fisher BUT there are drowsiness and brisk
reflexes.
Management:
1) Plasma exchange
2) IVIG (IV immunoglobulins) 0.5 g/kg/day for 5 days: as effective as plasma
exchange. No benefit in combining both treatments. IVIG may be easier to
administer and tends to have fewer side-effects.
Steroids and immunosuppressants have not been shown to be beneficial.
Dr Khaled Magraby MRCP Notes Neurology 1036
Chapter 10: Neurology
FVC (Forced vital capacity) regularly to monitor respiratory function.
FVC is also the best way to monitor respiratory function in any neurological disorders
that can affect the respiratory muscles (e.g. GBS, myasthenia gravis) and a better
indication of the need for ventilation.
FVC is used to monitor respiratory function in GBS
Prognosis:
20% suffer permanent disability, 5% die
Poor prognostic features
1) Age > 40 years
2) Previous history of a diarrhoeal illness (specifically Campylobacter
jejuni)
3) High anti-GM1 antibody titre
4) Poor upper extremity muscle strength
5) Need for ventilatory support
6) There is currently contradictory evidence as to whether a gradual or
rapid onset of GBS is associated with a poor outcome.
Diaphragmatic weakness occurs in 30% of patients with patients with GBS and
involvement of the neck muscles, tongue and palate leads to further respiratory
compromise.
Respiratory muscle function is best monitored by frequent assessment of the forced
vital capacity (FVC).
ITU admission is recommended when FVC is less than 20 mL/kg and intubation is
recommended in most cases when FVC is less than 15 mL/kg.
EX: A female patient aged 30 has a 5 year history of difficulty getting upstairs and
out of a low chair and mild upper limb weakness but no pain. There is no family
history. She presented with severe type 2 respiratory failure. EMG showed
evidence of myopathy. Which is the most likely diagnosis?
Acid maltase deficiency typically presents with insidious onset of proximal
myopathy and early respiratory muscle weakness.
Dr Khaled Magraby MRCP Notes Neurology 1037
Chapter 10: Neurology
NB: Botulism: have the same clinical presentation of descending weakness (i.e.
opposite in direction of GBS) with autonomic dysfunction (fixed dilated pupils).
It is a neuromuscular junction disorder and therefore nerve conduction studies
and EMG are normal.
Repetitive nerve stimulation shows incremental responses, is diagnostic of
botulism.
NB: CIDP (Chronic inflammatory demyelinating polyneuropathy) is clinically
similar to GBS (hyporeflexia or areflexia, paraesthesia and mild sensory deficits in
the upper and lower extremities, weakness) except that it follows a chronic
progressive course.
Meralgia paraesthetica
Caused by compression of lateral cutaneous nerve of thigh.
Typically burning sensation over antero-lateral aspect of thigh.
Burning thigh pain >>? Meralgia paraesthetica >> lateral cutaneous nerve of thigh
compression.
Neuropathic pain
Neuropathic pain may be defined as pain which arises following damage or
disruption of the nervous system.
It is often difficult to treat and responds poorly to standard analgesia.
Examples include:
1) Diabetic neuropathy
2) Post-herpetic neuralgia
3) Trigeminal neuralgia
4) Prolapsed intervertebral disc
NICE issued guidance in 2010 on the management of neuropathic pain:
First-line treatment*: oral Amitriptyline or Pregabalin.
If satisfactory pain reduction is obtained with amitriptyline but the person
cannot tolerate the adverse effects (it is better to avoid amitriptyline in pt. with
Dr Khaled Magraby MRCP Notes Neurology 1038
Chapter 10: Neurology
BPH due to the risk of urinary retention), consider oral imipramine or
nortriptyline as an alternative.
Second-line treatment: if first-line treatment was with amitriptyline, switch to or
combine with pregabalin. If first-line treatment was with pregabalin, switch to
or combine with amitriptyline.
Other options: pain management clinic, tramadol (no other strong opioids),
topical lidocaine for localised pain if patients unable to take oral medication.
*Please note that for some specific conditions the guidance may vary. For example:
Duloxetine for diabetic neuropathy.
Carbamazepine is used first-line for trigeminal neuralgia.
Trigeminal neuralgia
Trigeminal neuralgia is a pain syndrome characterised by severe unilateral pain. The
vast majority of cases are idiopathic but compression of the trigeminal roots by
tumours or vascular problems may occur.
The International Headache Society defines trigeminal neuralgia as:
A unilateral disorder characterised by brief electric shock-like pains, abrupt in
onset and termination, limited to one or more divisions of the trigeminal nerve.
The pain is commonly evoked by light touch, including washing, shaving,
smoking, talking, and brushing the teeth (trigger factors), and frequently
occurs spontaneously.
Small areas in the nasolabial fold or chin may be particularly susceptible to
the precipitation of pain (trigger areas).
The pains usually remit for variable periods.
Management:
Carbamazepine is first-line*
Failure to respond to treatment or atypical features (e.g. < 50 years old)
should prompt referral to neurology.
*The 2010 NICE neuropathic pain guidelines recommend using amitriptyline or
pregabalin first-line for non-diabetic neuropathic pain, but makes no specific
recommendation for trigeminal neuralgia.
Due to the amount of evidence supporting carbamazepine in trigeminal neuralgia
and its recommendation in consensus guidelines (including Clinical Knowledge
Summaries) the author does not feel that this recommendation should be changed
for now.
Dr Khaled Magraby MRCP Notes Neurology 1039
Chapter 10: Neurology
Peripheral neuropathy (PN): demyelinating vs. axonal
Axonal pathology:
1) Alcohol
2) DM (± a demyelinating picture)
3) Vit B12 deficiency (± a demyelinating picture)
4) Vasculitis
5) Renal failure.
6) HSMN type II
Demyelinating pathology:
1) GBS (Guillain-Barre syndrome)
2) CIDP (Chronic inflammatory demyelinating polyneuropathy).
3) Paraprotein neuropathy
4) Amiodarone
5) HSMN type I
Hereditary Sensorimotor Neuropathy (HSMN)
It is a relatively new term which encompasses Charcot-Marie-Tooth disease (also
known as perineal muscular atrophy).
Over 7 types have been characterised - however only 2 are common clinically:
HSMN type I: primarily due to demyelinating pathology
HSMN type II: primarily due to axonal pathology
HSMN type I:
Autosomal dominant, so 50% of children will be affected.
Due to defect in PMP-22 gene (which codes for myelin).
Features often start at puberty.
Motor symptoms predominate.
Distal muscle wasting, pes cavus (high arched foot), clawed toes.
Foot drop, leg weakness often first features.
Nerve Conduction Velocity (NCV) < 30 m/sec.
Dr Khaled Magraby MRCP Notes Neurology 1040
Chapter 10: Neurology
Nerve conduction studies (NCS)
It is useful in determining between axonal and demyelinating pathology
Axonal Demyelinating
Normal conduction velocity Reduced conduction velocity
Reduced amplitude Normal amplitude
Axonal Demyelinating
Conduction velocity Normal Reduced
Amplitude Reduced Normal
Peripheral neuropathy
It may be divided into conditions which predominately cause a motor or sensory loss.
Predominately motor loss Predominately sensory loss:
GBS. DM.
CIDP. Alcoholism.
HSMN Vit B12 deficiency.
Porphyria. Uraemia.
Amyloidosis.
Lead poisoning
Leprosy.
Diphtheria
Alcoholic neuropathy:
Secondary to both direct toxic effects and reduced absorption of Vit. B.
Sensory symptoms typically present prior to motor symptoms.
Vitamin B12 deficiency:
Subacute combined degeneration (SCD) of spinal cord.
Dorsal column usually affected first (joint position, vibration) prior to distal
paraesthesia.
Dr Khaled Magraby MRCP Notes Neurology 1041
Chapter 10: Neurology
Drugs causing peripheral neuropathy
Drugs causing a peripheral neuropathy:
1) Antibiotics: Nitrofurantoin, Metronidazole
2) Amiodarone
3) Isoniazid
4) Vincristine & most chemotherapy
5) TCA (Tricyclic antidepressants)
Autonomic neuropathy
Features:
Impotence,
Urinary retention.
Postural hypotension e.g. drop of 30/15 mmHg
Inability to sweat
Loss of decrease in heart rate following deep breathing
Pupils: dilates following adrenaline instillation
Causes:
DM
GBS (Guillain-Barre syndrome)
Parkinson's
MSA (Multisystem atrophy)(Shy-Dragger syndrome)
Infections: HIV, Chagas' disease, neurosyphilis
Drugs: antihypertensives, TCA
Craniopharyngioma
Dr Khaled Magraby MRCP Notes Neurology 1042
Chapter 10: Neurology
Carpal tunnel syndrome
It is caused by compression of median nerve in the carpal tunnel.
History:
Pain/pins and needles in thumb, index, middle finger
Unusually the symptoms may 'ascend' proximally
Classically at night
Patient shakes his hand to obtain relief.
Examination:
Weakness of thumb abduction (abductor pollicis brevis)
Wasting of thenar eminence (NOT hypothenar)
Tinel's sign: Tapping causes paraesthesia
Phalen's sign: flexion of wrist causes symptoms
The median nerve also supplies most of the muscles of the thenar eminence,
namely the (LOAF):
1) Lateral two lumbricals
2) Opponens pollicis
3) Abductor pollicis brevis, and
4) Flexor pollicis brevis
NB: The remaining small muscles of the hand are supplied by the ulnar nerve.
Causes:
Idiopathic
Pregnancy
Oedema e.g. heart failure
Lunate fracture
Rheumatoid arthritis
Electrophysiology:
Motor + sensory: prolongation of the action potential
Treatment:
1) Corticosteroid injection
2) Wrist splints at night
3) Surgical decompression (flexor retinaculum division)
Dr Khaled Magraby MRCP Notes Neurology 1043
Chapter 10: Neurology
NB: The palmar cutaneous branch of the median nerve lies superficial to the
flexor retinaculum and does not pass through the carpal tunnel. It supplies the skin
over the thenar eminence, which is therefore spared in carpal tunnel syndrome.
So Pt. with CTS have preserved skin sensation over thenar area.
Ulnar nerve
It arises from medial cord of brachial plexus (C8, T1)
The ulnar nerve supplies all the muscles of the hands except thenar muscles and
the lateral two lumbricals which are supplied by median nerve.
Motor: to
Medial two lumbricals
aDductor pollicis >> Adduction of the thumb
Interossei
Hypothenar muscles: (opponens digiti minimi, abductor digiti minimi, flexor
digiti minimi).
Flexor carpi ulnaris
Sensory to:
Medial 1 1/2 fingers (palmar and dorsal aspects)
Patterns of damage:
Damage at wrist:
'Claw hand' - hyperextension of the metacarpophalangeal joints and flexion
at the distal and proximal interphalangeal joints of the 4th and 5th digits
Wasting and paralysis of intrinsic hand muscles (except lateral two lumbricals)
Wasting and paralysis of hypothenar muscles
Sensory loss to the medial 1 1/2 fingers (palmar and dorsal aspects)
Damage at elbow:
As above (however, ulnar paradox - clawing is more severe in distal lesions)
Radial deviation of wrist
Dr Khaled Magraby MRCP Notes Neurology 1044
Chapter 10: Neurology
Paraneoplastic syndromes affecting nervous system
1) Lambert-Eaton Myasthenic syndrome:
Associated with small cell lung cancer (also breast and ovarian).
Antibody directed against pre-synaptic voltage gated calcium channel in the
peripheral nervous system.
Can also occur independently as autoimmune disorder.
2) Anti-Hu: (imagine H sticks as 2 lungs or 2 brain hemispheres)
Associated with small cell lung carcinoma and neuroblastoma.
Sensory neuropathy - may be painful
Cerebellar syndrome
Encephalomyelitis
3) Anti-Yo: (imagine Y as lady’s private organs)
Associated with ovarian and breast cancer
Cerebellar syndrome
4) Anti-GAD antibody:
Associated with breast, colorectal and small cell lung carcinoma
Stiff person's syndrome or diffuse hypertonia.
5) Anti-Ri:
Associated with breast and small cell lung carcinoma
Ocular opsoclonus-myoclonus
EX: A 66-year-old woman is investigated for ascites and found to have ovarian
cancer. She presents due to 'unsteadiness'. On examination there is evidence of
nystagmus and past-pointing. Which one of the following antibodies is most likely to
be present? >>> Anti-Yo.
EX: A 65-year-old man who is known to have metastatic colorectal cancer presents
for review. Since last been seen he reports being generally stiff and on examination
is noted to have diffuse hypertonia. Which antibodies are most likely to be
responsible for this presentation? >>> Anti-GAD.
Dr Khaled Magraby MRCP Notes Neurology 1045
Chapter 10: Neurology
Mitochondrial diseases
Whilst most DNA is found in the cell nucleus, a small amount of double-stranded
DNA is present in the mitochondria.
It encodes protein components of the respiratory chain and some special types of
RNA.
Mitochondrial inheritance has the following characteristics:
Inheritance is only via the maternal line as the sperm contributes no
cytoplasm to the zygote.
All children of affected males will not inherit the disease
All children of affected females will inherit it
Generally encode rare neurological diseases
Poor genotype: phenotype correlation - within a tissue or cell there can be
different mitochondrial populations - this is known as heteroplasmy.
Histology:
Muscle biopsy classically shows 'red, ragged fibres' due to increased
number of mitochondria.
Examples include:
1) Leber's optic atrophy: bilateral painless visual loss in young boy.
2) MELAS syndrome: Mitochondrial Encephalomyopathy Lactic Acidosis and
Stroke-like episodes.
3) MERRF syndrome: Myoclonus Epilepsy with Ragged-Red Fibres.
4) Kearns-Sayre syndrome: onset in patients < 20 years old, external
ophthalmoplegia, retinitis pigmentosa. Bilateral ptosis may be seen.
5) Sensorineural hearing loss
Dr Khaled Magraby MRCP Notes Neurology 1046
Chapter 10: Neurology
Prescribing in pregnant patients
Very few drugs are known to be completely safe in pregnancy.
The list below largely comprises of those known to be harmful.
Some countries have developed a grading system.
Antibiotics:
Tetracyclines
Aminoglycosides
Sulphonamides and trimethoprim
Quinolones: the BNF advises to avoid due to arthropathy in some animal
studies
Other drugs:
ACEI and ARBS
Statins
Warfarin
Sulfonylureas
Retinoid (including topical)
Cytotoxic agents
The majority of antiepileptics including valproate, carbamazepine and phenytoin are
known to be potentially harmful.
The decision to stop such treatments however is difficult as uncontrolled epilepsy is
also a risk
Pituitary tumours
Hormones secreted:
Prolactin- 35%
No obvious hormone, 'non-functioning', 'chromophobe' - 30%
Growth hormone - 20%
Prolactin and growth hormone - 7%
ACTH - 7%
Others: TSH, LH, FSH - 1%
Dr Khaled Magraby MRCP Notes Neurology 1047
Chapter 10: Neurology
NB: Prolactinomas are unusual as medical therapy is first line, even if visual field
defects are present.
The main indications for surgery are tumours resistant to dopamine agonists.
Prolactinoma management >> Medical therapy (dopamine agonist) is almost
always first-line, even if visual field defects are present.
Transient global amnesia (TGA)
It represents a transient vascular insufficiency of both hippocampi.
This is a rare condition which usually affects people over the age of 50.
It lasts less than 24 hours and an awareness of personal identity is retained,
along with normal cognition.
The cause is unknown but it may be related to a migrainous phenomenon or
transient ischaemia.
It is claimed that the condition may be associated with neuronal loss in the
hippocampal area or abnormal metabolism by neurones in this area leading to build
up of lactate, but definitive proof does not exist.
TTT is Reassurance.
No specific therapy is required; specifically no increased use of anti-platelet agents
is needed.
EX: A 60-year-old man presents with an episode of memory loss.
Three days earlier he had become confused. His wife led him into the house - he
apparently sat down at her request, and had a cup of tea. He then wandered
around the house, confused, but remained conscious and able to have some
conversation with his wife, though continuing to ask similar questions
repeatedly.
After 3 hours, he abruptly returned to normal and had no recollection of the
events.
Dr Khaled Magraby MRCP Notes Neurology 1048
Chapter 10: Neurology
Sciatic nerve palsy
Sciatic nerve palsy is a known complication of a total hip replacement (femoral
nerve palsy can occur but is much less common).
It causes global weakness of the ankle due to the involvement of both of its
branches: tibial nerve (plantaflexion and inversion) and common peroneal
nerve (dorsiflexion and eversion).
The ankle jerk is absent due to tibial nerve involvement.
When back pain is caused by a L5/S1 disc prolapse, the S1 nerve root may be
affected
The Achilles reflex tests the S1/2 nerve root, and sciatic nerve.
NB: Ankle reflex (tibial nerve mediated) and
NB: Knee reflex (femoral nerve mediated).
NB: Foot drop >>> affection of common peroneal nerve
Conus medularis syndrome
Conus medularis syndrome presents with mixed UMNL and LMNL signs.
These include bilateral distal weakness with increased tone and hyper-reflexia,
fasciculation. Sensory loss is most marked in the perianal region. It is much rarer
that cauda equina syndrome.
Cauda equina syndrome is associated with flaccid paralysis.
Spinal cord infarction related to anterior spinal artery disease presents most
frequently with sudden onset pain and loss of power and sensation beginning in the
thoracic region. The symptoms seen here have slowly built up over a few weeks and
affect a more distal spinal cord distribution.
Sacroiliitis would not be associated with neurological deficit.
Dr Khaled Magraby MRCP Notes Neurology 1049
Chapter 10: Neurology
Conus Medullaris
Cauda Equina Syndrome
Syndrome
Gradual and may be unilateral
Presentation Sudden and bilateral
leg signs initially
Knee jerks preserved, ankle Both knee and ankle jerks
Reflexes
jerks affected affected
Radicular
Less severe More severe
pain
Numbness often localised to Numbness often localised to the
perianal area, dissociation saddle area, may be
Sensory
can occur, usually bilateral asymmetrical and unilateral,
and symmetrical sensory loss often dermatomal
Symmetrical, hyperreflexic Areflexic paraplegic, may be
disal paresis, less marked asymmetric, more marked than
Motor
than cauda equina, may be conus medullaris, fasciculations
fasciculations rare, atrophy more common
Impotence Frequent Often less marked
Urinary retention and atonic
Urinary retention, usually
Sphincter anal sphincter present early in
presents later in course of
dysfunction disease (can cause overflow
disease
urinary incontinence)
Low back
More marked Less marked
pain
Dr Khaled Magraby MRCP Notes Neurology 1050
Chapter 10: Neurology
Glasgow coma scale (GCS)
Eye opening Verbal response Motor response
(4) (5) (6)
Spontaneously Orientated Obeys verbal commands
To speech Disorientated Localises painful stimuli
To painful stimulus Inappropriate words Withdrawal to pain
No response Incomprehensible sounds Flexion to pain
No response Extension to pain
No response
GCS is meaningless unless it is broken down into its components (E V M).
It is important to note that the GCS is unreliable and should not be applied to
patients who are inebriated, intubated or who have a therapeutic or traumatic
paralysis.
Miscellaneous:
Gait disturbance may occur for a variety of reasons:
1) Sensory ataxia in B12 deficiency and tabes dorsalis
2) Pyramidal signs in B12 deficiency and SLE
3) Cerebellar ataxia in cerebrotendinous xanthomatosis.
Aphasia:
- In Broca's or non-fluent or expressive aphasia the patient is unable to name
objects with poor comprehension and repetition. It localises to the Broca's area in the
left posterior inferior frontal gyrus.
- In Wernicke's or fluent or receptive aphasia the patient is able to form correct
grammatical sentences but language content is incorrect. It localises to the dominant
superior temporal gyrus. There is poor comprehension and repetition but verbal
output is fluent.
Dr Khaled Magraby MRCP Notes Neurology 1051
Chapter 10: Neurology
- Transcortical motor aphasia which localises to the anterior superior frontal lobe,
the patient is able to repeat and have good comprehension and repetition but is
unable to express themselves and have halting, effortful speech with intact
repetition, Patients also have impaired writing skills.
- Transcortical sensory aphasia has impaired auditory comprehension with intact
repetition and fluent speech, but the patient is unable to follow verbal commands
with fluent grammatical speech. It differs from Wernicke's aphasia in that patients still
have intact repetition, and exhibit cholalia (the compulsive repetition of words).
- Conduction aphasia. It is characterised by frequent speech errors, impaired
repetition, and reduced phonological short-term memory and naming difficulties. In
contrast to other forms of dysphasia, speech output is otherwise fluent and
grammatically correct. Comprehension is intact.
- Anomic aphasia (also known as nominal aphasia) results in word finding
difficulties. On closer examination there may also be repetition problems and
comprehension problems but these are typically mild compared to other aphasia
syndromes.
- Global aphasia results in an almost mute patient: there is poor verbal output,
comprehension, repetition and understanding. With deficits in all aspects of
language: spontaneous speech, naming, repetition, auditory comprehension, reading
and writing. It is commonly seen in patients with large infarctions of the left cerebral
hemisphere, usually due to occlusion of the internal carotid or middle cerebral artery.
The visual centres remain intact and therefore patients are able to follow instructions
shown to them.
- Aphemia is a type of aphasia in which there is severe dysarthria and impairment of
verbal output. There is intact comprehension. It is believed to be the result of pars
opercularis, inferior pre-Rolandic gyrus or subcortical lesions.
Spaces:
- Subarachnoid space is in between the arachnoid mater and the pia mater.
Vascular malformations and aneurysms typically bleed in the subarachnoid space
e.g. CT shows blood in the Sylvain fissure.
- Subdural refers to the area between the dura and the arachnoid.
- Epidural is between the skull and the dura.
- Subgaleal is a potential space between the skull and the scalp aponeurosis.
- Subcortical is in the white matter of the brain below the cortex.
Dr Khaled Magraby MRCP Notes Neurology 1052
Chapter 10: Neurology
Lesions:
- FTD (fronto-temporal dementia) appears to begin in the orbitofrontal cortex and
anterior cingulate regions of the frontal lobes, along with the anterior insula.
- Damage to the hippocampus and parahippocampus results in memory problems
and has early involvement in Alzheimer's disease.
- The corpus callosum can be involved in multiple sclerosis where so-called
Dawson's fingers can be seen.
- Prefrontal cortex damage can result in disinhibition and problems with social
interaction and judgement and has been implicated in schizophrenia.
The differential diagnoses in a patient presenting with headaches and painful
diplopia are:
1) A posterior communicating aneurysm (PCA).
2) Ophthalmoplegic migraine.
3) Pituitary adenoma/ Apoplexy.
4) Cavernous sinus thrombosis, or
5) A medical mononeuritis.
Conventional angiography is the definitive procedure for the detection and
characterisation of cerebral aneurysms.
Digital subtraction angiography may be helpful in identifying an acutely ruptured
aneurysm.
Dr Khaled Magraby MRCP Notes Neurology 1053